Plausible hypothesis on why gravity might fail on earth

The name of the pictureThe name of the pictureThe name of the pictureClash Royale CLAN TAG#URR8PPP











up vote
14
down vote

favorite
6












I've been grappling with a story for a while, involving a scenario where gravity starts to fail. The goal of the story is mostly how people react to the knowledge that the world will end. But, I also wanted to include some sort of scientific group that is trying to figure out why. Are there plausible reasons that gravity would fail? I like stories to have at least a pinch of truth as an anchor, and I do want some sort of thought process that scientists would go through. Obviously there would be talking heads on TV giving reasons, so, what would those reasons be.







share|improve this question
















  • 1




    you might want to check out The Age of Mircles by Karen Thompson Walker which covers the same theme you are pursuing.
    – Henry Taylor
    Aug 9 at 19:21










  • Welcome to Worldbuilding.SE! We're glad you could join us! When you have a moment, please click here to learn more about our culture and take our tour. Thanks!
    – JBH
    Aug 9 at 19:24






  • 5




    "Obviously there would be talking heads on TV giving reasons" Not for long. Gravity failing would radically alter, among other things, the Sun, the orbits of planets (we'd drift out into space in a larger and larger orbit - possibly an escape trajectory and freeze in short order). The atmosphere would reduce in surface pressure and would likely start to escape into space (but we'd freeze long before that happened). Oceans and pressure from Earth's core would imply an expansion of the crust (really bad). The list of ways even a small change in gravity would kills us is alarming.
    – StephenG
    Aug 10 at 2:22






  • 2




    Required watching?: Patema Inverted
    – Clockwork-Muse
    Aug 10 at 17:33






  • 1




    A rogue planet enters into orbit around the Earth. The two gravities cancel eachother out. The world ends when the two collide. It's not an original idea but it's accurate.
    – cgTag
    Aug 11 at 1:35














up vote
14
down vote

favorite
6












I've been grappling with a story for a while, involving a scenario where gravity starts to fail. The goal of the story is mostly how people react to the knowledge that the world will end. But, I also wanted to include some sort of scientific group that is trying to figure out why. Are there plausible reasons that gravity would fail? I like stories to have at least a pinch of truth as an anchor, and I do want some sort of thought process that scientists would go through. Obviously there would be talking heads on TV giving reasons, so, what would those reasons be.







share|improve this question
















  • 1




    you might want to check out The Age of Mircles by Karen Thompson Walker which covers the same theme you are pursuing.
    – Henry Taylor
    Aug 9 at 19:21










  • Welcome to Worldbuilding.SE! We're glad you could join us! When you have a moment, please click here to learn more about our culture and take our tour. Thanks!
    – JBH
    Aug 9 at 19:24






  • 5




    "Obviously there would be talking heads on TV giving reasons" Not for long. Gravity failing would radically alter, among other things, the Sun, the orbits of planets (we'd drift out into space in a larger and larger orbit - possibly an escape trajectory and freeze in short order). The atmosphere would reduce in surface pressure and would likely start to escape into space (but we'd freeze long before that happened). Oceans and pressure from Earth's core would imply an expansion of the crust (really bad). The list of ways even a small change in gravity would kills us is alarming.
    – StephenG
    Aug 10 at 2:22






  • 2




    Required watching?: Patema Inverted
    – Clockwork-Muse
    Aug 10 at 17:33






  • 1




    A rogue planet enters into orbit around the Earth. The two gravities cancel eachother out. The world ends when the two collide. It's not an original idea but it's accurate.
    – cgTag
    Aug 11 at 1:35












up vote
14
down vote

favorite
6









up vote
14
down vote

favorite
6






6





I've been grappling with a story for a while, involving a scenario where gravity starts to fail. The goal of the story is mostly how people react to the knowledge that the world will end. But, I also wanted to include some sort of scientific group that is trying to figure out why. Are there plausible reasons that gravity would fail? I like stories to have at least a pinch of truth as an anchor, and I do want some sort of thought process that scientists would go through. Obviously there would be talking heads on TV giving reasons, so, what would those reasons be.







share|improve this question












I've been grappling with a story for a while, involving a scenario where gravity starts to fail. The goal of the story is mostly how people react to the knowledge that the world will end. But, I also wanted to include some sort of scientific group that is trying to figure out why. Are there plausible reasons that gravity would fail? I like stories to have at least a pinch of truth as an anchor, and I do want some sort of thought process that scientists would go through. Obviously there would be talking heads on TV giving reasons, so, what would those reasons be.









share|improve this question











share|improve this question




share|improve this question










asked Aug 9 at 19:06









SEK1977

16318




16318







  • 1




    you might want to check out The Age of Mircles by Karen Thompson Walker which covers the same theme you are pursuing.
    – Henry Taylor
    Aug 9 at 19:21










  • Welcome to Worldbuilding.SE! We're glad you could join us! When you have a moment, please click here to learn more about our culture and take our tour. Thanks!
    – JBH
    Aug 9 at 19:24






  • 5




    "Obviously there would be talking heads on TV giving reasons" Not for long. Gravity failing would radically alter, among other things, the Sun, the orbits of planets (we'd drift out into space in a larger and larger orbit - possibly an escape trajectory and freeze in short order). The atmosphere would reduce in surface pressure and would likely start to escape into space (but we'd freeze long before that happened). Oceans and pressure from Earth's core would imply an expansion of the crust (really bad). The list of ways even a small change in gravity would kills us is alarming.
    – StephenG
    Aug 10 at 2:22






  • 2




    Required watching?: Patema Inverted
    – Clockwork-Muse
    Aug 10 at 17:33






  • 1




    A rogue planet enters into orbit around the Earth. The two gravities cancel eachother out. The world ends when the two collide. It's not an original idea but it's accurate.
    – cgTag
    Aug 11 at 1:35












  • 1




    you might want to check out The Age of Mircles by Karen Thompson Walker which covers the same theme you are pursuing.
    – Henry Taylor
    Aug 9 at 19:21










  • Welcome to Worldbuilding.SE! We're glad you could join us! When you have a moment, please click here to learn more about our culture and take our tour. Thanks!
    – JBH
    Aug 9 at 19:24






  • 5




    "Obviously there would be talking heads on TV giving reasons" Not for long. Gravity failing would radically alter, among other things, the Sun, the orbits of planets (we'd drift out into space in a larger and larger orbit - possibly an escape trajectory and freeze in short order). The atmosphere would reduce in surface pressure and would likely start to escape into space (but we'd freeze long before that happened). Oceans and pressure from Earth's core would imply an expansion of the crust (really bad). The list of ways even a small change in gravity would kills us is alarming.
    – StephenG
    Aug 10 at 2:22






  • 2




    Required watching?: Patema Inverted
    – Clockwork-Muse
    Aug 10 at 17:33






  • 1




    A rogue planet enters into orbit around the Earth. The two gravities cancel eachother out. The world ends when the two collide. It's not an original idea but it's accurate.
    – cgTag
    Aug 11 at 1:35







1




1




you might want to check out The Age of Mircles by Karen Thompson Walker which covers the same theme you are pursuing.
– Henry Taylor
Aug 9 at 19:21




you might want to check out The Age of Mircles by Karen Thompson Walker which covers the same theme you are pursuing.
– Henry Taylor
Aug 9 at 19:21












Welcome to Worldbuilding.SE! We're glad you could join us! When you have a moment, please click here to learn more about our culture and take our tour. Thanks!
– JBH
Aug 9 at 19:24




Welcome to Worldbuilding.SE! We're glad you could join us! When you have a moment, please click here to learn more about our culture and take our tour. Thanks!
– JBH
Aug 9 at 19:24




5




5




"Obviously there would be talking heads on TV giving reasons" Not for long. Gravity failing would radically alter, among other things, the Sun, the orbits of planets (we'd drift out into space in a larger and larger orbit - possibly an escape trajectory and freeze in short order). The atmosphere would reduce in surface pressure and would likely start to escape into space (but we'd freeze long before that happened). Oceans and pressure from Earth's core would imply an expansion of the crust (really bad). The list of ways even a small change in gravity would kills us is alarming.
– StephenG
Aug 10 at 2:22




"Obviously there would be talking heads on TV giving reasons" Not for long. Gravity failing would radically alter, among other things, the Sun, the orbits of planets (we'd drift out into space in a larger and larger orbit - possibly an escape trajectory and freeze in short order). The atmosphere would reduce in surface pressure and would likely start to escape into space (but we'd freeze long before that happened). Oceans and pressure from Earth's core would imply an expansion of the crust (really bad). The list of ways even a small change in gravity would kills us is alarming.
– StephenG
Aug 10 at 2:22




2




2




Required watching?: Patema Inverted
– Clockwork-Muse
Aug 10 at 17:33




Required watching?: Patema Inverted
– Clockwork-Muse
Aug 10 at 17:33




1




1




A rogue planet enters into orbit around the Earth. The two gravities cancel eachother out. The world ends when the two collide. It's not an original idea but it's accurate.
– cgTag
Aug 11 at 1:35




A rogue planet enters into orbit around the Earth. The two gravities cancel eachother out. The world ends when the two collide. It's not an original idea but it's accurate.
– cgTag
Aug 11 at 1:35










22 Answers
22






active

oldest

votes

















up vote
3
down vote



accepted










Dark Energy / The Big Rip



There is a possibility that this will eventually happen. It depends on the energy density of dark energy in the universe, which we are currently unable to measure accurately enough to tell.



Strictly speaking, this is not gravity failing so much as the expansion of space accelerating, but for all intents and purposes the results are the same. Basically everything in the universe is accelerating away from everything else, even on tiny scales, but the rate of expansion is greatly overpowered by gravity except on intergalactic scales. But if the expansion continues to accelerate, eventually it will overpower gravity.



This will actually create a predictable sequence of events that can happen on human-measurable scales. It could make a great setting for a frantic race against the clock as scientists struggle to figure out a way of escaping the universe before everything ends.



Using one hypothetical example using real equations, the Big Rip could happen 22 billion years from now. (This will be long after our own solar system is gone, but humanity's descendants could still be around).



60 million years before the end, galaxies become gravitationally unbound. Your setting is a lone solar system in a starless void. It is still habitable but the knowledge that there was once such a thing as stars could be long forgotten.



About 3 months before the end, the gravitational attraction between the planet and its star is overpowered by the expansion of space. The planet begins to move away from its star, first slowly and then picking up speed until people are forced to use technology to survive the cold. But things are going to get much worse...



Gravity starts to weaken. Quakes and cataclysmic upheaval become commonplace as the pressure holding the planet's insides inside diminishes. The atmosphere grows thin. The oceans boil. In the last few minutes before the end, the gravitational force cannot hold the planet together anymore and every planet in the solar system explodes at around the same time that the distant sun evaporates into the void.



Anyone who manages to survive in spacecrafts have only a few minutes left before the accelerating expansion of space overpowers the binding energy of matter itself. There might be a few moments where they can actually feel it building up inside them before it rips apart their bodies, then their molecules, atoms, and finally subatomic particles themselves.



You can adjust the timing if you want to introduce any kind of handwavium physical laws that we don't know about yet.






share|improve this answer



























    up vote
    39
    down vote













    Totally different idea.



    In late 2283, a research team at the University of New Tokyo finally cracked the Unified Field problem. This gained them one of the most prestigious Nobel Prizes ever awarded in history.



    The results of this were initially amazing: it allowed on demand gravity control, the benefits of which were immense. Needless to say, everyone started making gravity control devices for vehicle levitation, space exploration, construction, etc. You name it, we found an application for gravity control.



    However, what nobody had anticipated was that the count of particles moderating the forces in the unified field theorem had been a balanced set of parameters, and the changes that occurred as a result of gravity control led to an ever increasing instability.



    In essence we'd been living in a local minimum for this parameter equation where everything was stable, but gravity control eventually pushed us out of this local minimum into an incredibly unstable landscape. One of the major side effects of this was that gravity slowly, but irreversibly, started to decrease.






    share|improve this answer
















    • 21




      This almost sounds like an allegory for the Earth's dependence on oil. We become dependent on a specific resource and start to use it for everything everywhere. That same widespread deployment turns around and bites us. Good suggestion.
      – Ian Johnson
      Aug 10 at 3:02






    • 5




      I was just reading this answer thinking how did I not hear about this... then I reread the date!
      – Liath
      Aug 10 at 12:28










    • I like this answer because it provides your story with so many possibilities, the hubris and arrogance, and ignorance, of the society that has become dependent on technology. Possibly also an answer too to the Fermi Paradox :) A great filter indeed.
      – flox
      Aug 12 at 17:26

















    up vote
    25
    down vote













    We need to have more fun with this question



    Keep in mind that gravity is but one characteristic of mass. You can't change just gravity without side effects. Those side effects are valuable to you as they can be used to advance your plot.



    • A wormhole appears at the center of the planet, pulling mass away from the core. Thanks to the shell theorem, gravity must reduce to zero as you approach the center of the planetary sphere. Thus, the appearance wouldn't be immediately noticable, and if the mass were drawn off slowly enough, it wouldn't be appreciably obvious. Note that there would be side effects, such as liquid core currents changing, which would change the magnetosphere and possibly plate tectonics.


    • Something in the universe, some ribbon of effect we know nothing about, is passing through our solar system and, thus, our planet is passing through it. This ribbon is changing the positive charge strength forcing atomic nucleai to spread apart. Not only is this reducing the density of planetary mass (and therefore gravity), but it's also causing the planet to increase in circumference.


    • Something unseen is passing through the solar system. A "dark planet" (or, perhaps of greater interest, a ribbon of dark asteroids) with sufficient mass as any point of its passing to change the effective gravity on Earth. The planet or ribbon can pass through on a parabolic arc such that it would affect the Earth for most of a year. The side effect is that Earthlings standing on the side closest to the dark planet/belt/ribbon would be lighter, while those on the opposite side would feel heavier. Earthquakes would be normal. Shifting the Earth's orbit (perhaps moving it further from the sun, thereby solving global warming) would be expected.


    • Q is really bored and decides to change the gravitational constant of the universe — but only in the Sol system. Not only would this be really hard to figure out what was going on, but ships leaving the system would sail through a breakwater (I'm using the wrong word, that point in the ocean when leaving a beach when the undersea landmass drops away and you get serious wave action. It killed the best friend of Moana's father. It was really sad and should convince all to live in Iowa).


    • The theory that you can treat gravity as coming from a point source at the center of a sphere is only true when the sphere is reasonably solid or homogenous. Let's have some reaction (let's blame it on the Somalis this time, they built a planetary core implosion device) that caused the inner 25% of the planet's volume to suddenly compress such that a massive void appeared between the new basically neutronium core and the rest of the core/mantle. Not only would this change gravity by pulling a lot of mass further away from the surface, but it would have the side effect of the planet slowly starting to wobble in its rotation until that big marble comes in contact with the rest of the core and begins acting like that child's tooth in a paint can... or until the sudden vacuum causes the planet to violently shrink.


    What else, my fellow WorldBuildians! In what other ways can be bend, contort, graffiti, or otherwise ignore the fundamental rules of orbital mechanics and classical physics to bring about, if nothing else, the appearance of a lessening gravity?






    share|improve this answer
















    • 3




      You forgot to pay the gravity bill... goo.gl/images/h8NWJN
      – Jim Garrison
      Aug 10 at 5:12











    • This sounds like the most plausible (or, least implausible) of the existing answers as I write this, according to current science.
      – David Z
      Aug 10 at 5:45










    • Is the term you’re looking for “shelf break” or “continental slope?” (source)
      – J F
      Aug 10 at 10:57










    • +1 shell theory
      – PV22
      Aug 11 at 17:47

















    up vote
    17
    down vote













    The Gravitational Constant is not uniform throughout space and time



    Scientists have long speculated that some fundamental constants of the universe, like the speed of light or the fine structure constants, are varying through time.



    Why not make it varying through space as well? This way a part of the galaxy can decay while the other parts are still bound by normal gravity.






    share|improve this answer
















    • 2




      Notably, if the parameters vary through time, you've lost conservation of energy. If they vary across space, you've lost conservation of momentum. (Noether's Theorem means every conservation law is equivalent to a symmetry; if you lose one you lose the other.)
      – Draconis
      Aug 11 at 21:18

















    up vote
    9
    down vote













    Unexpectedly for all the community of scientists in the world, gravitons (the particles mediating gravitational force) have a finite life time, and they are starting to decay.



    The more they decay, the more gravity is weakened. After a certain time gravity will no longer bind together masses, so unless they are electrically charged and can rely on good old Coulomb force, they are bound to separate.



    Space will be filled with atoms traveling around, no longer aggregated.






    share|improve this answer




















    • This is a VERY interesting way of looking at it!
      – JBH
      Aug 9 at 19:53










    • Also deeply depressing for the story, because the result would be inescapable. Kind of an On the Beach ending. "And in the universe, there was silence."
      – jdunlop
      Aug 9 at 20:07






    • 7




      "gravitons have a finite life time" - usually the particle life time is measured from its creation, and decaying force carriers would only manifest as a force that decays more than quadratically with distance. You'd need the life time to change over time. Do note that real-life gravity seems to have no such super-quadratic dropoff - and increasing it would crank up the expansion of the universe rather significantly. And in any case, we'd drift away from our Sun as a planet far before we'd drift away from our planet.as individual organisms, even without the increased expansion...
      – John Dvorak
      Aug 9 at 22:43










    • @JohnDvorak Perhaps the gravity particles act more like air does for sound — carrying it, but not being created when gravitational force is emitted. However, all the particles (that I know of) we’ve encountered have decayed exponentially, making this seem somewhat unrealistic to me.
      – J F
      Aug 10 at 11:00

















    up vote
    6
    down vote













    In order of decreasing viability (all of which are still super low):



    • The simulation hypothesis, that we are all living in a simulation, is really your best bet here. If gravity were capable of failing one would expect to see stars sort of just blinking out of existence every now and then, which we don't. That being said if we are living in a simulation (which is quite likely) then really the simulators can do whatever they want with physics.


    • Aliens, "pull the fabric of space-time taught." does it mean anything? not really no. is it intuitive? yes.


    • Act of god. plain simple, always in style. I'd go with this explaination myself


    • Freak variation in the gravitational constant. https://en.wikipedia.org/wiki/Time-variation_of_fundamental_constants. it's not certain that G the graviational constant of the universe is always well constant, Some theories hold that it varies. If it suddenly varies down (possibly even as a result of human tampering, probably not though.) gravity turns "off" simple as that. that being said this wipes out the universe not just earth. Bare in mind suddenly in this context could still be upwards of a million years or just a few seconds so it gives you a lot of narrative freedom.


    • Presence of large numbers of "Anti-Gravitons". the Graviton (a hypothetical particle that carries the force of gravity) sometimes has an even more hypothetical counter part called the anti graviton aleins fire a bunch of theese at us and they react in unexpected ways with currently existing gravitons destroying them and sending the excess energy away as gravitational waves.


    • Vacuum decay? there is a thing called Vacuum decay, most likely it will simply wipe out everything if real however there is a INCREDIBLY small chance that it simply re-writes the laws of reality instead.


    All of these answers (except Act of god & simulation, which have their own problems) are pseudo-scientific to the extreme and highly unlikely to actually work. If you go for simulation, god, or either of the aliens options you can always also put gravity back before the earth if fully destroyed leaving people left on earth with just a bunch of earth-quakes and a new outlook on life to deal with, if that's you narrative intention that is.






    share|improve this answer






















    • I haven't fully decided on an ending, so anything is possible... obviously life as everyone would know it would end... I have toyed with aliens or dimensional sources, but mostly as a way to narrate. As if all the story lines I'm presenting are part of a massive archeological find, and some being, whether an alien civilization or humans that somehow escape come back to piece together events.
      – SEK1977
      Aug 9 at 20:25






    • 2




      Do you mean pull it "taut"?
      – Azor Ahai
      Aug 9 at 23:17










    • Well of course i'm speaking nonsense when i say "pull it taut" however there is a demonstration of the behaviour of gravity where you place two balls on rubber sheet and they move together in this demonstration pulling the sheet taut will cause gravity to cease existing. of course in reality it's meaningless and more or less impossible but it makes more plausible explanation than a lot of stuff in sci-fi. in closest real world terms it would be the equivalent of using artificial gravity to exactly cancel out earths gravity
      – Ummdustry
      Aug 10 at 7:26

















    up vote
    5
    down vote













    It sounds like the big rip theory answers your question. Space-time expands and therefore the entire universe expands. The theory says the expansion will eventually overcome gravity.
    https://en.m.wikipedia.org/wiki/Big_Rip






    share|improve this answer




















    • I hadn't heard of this one, thank you.
      – SEK1977
      Aug 9 at 20:28

















    up vote
    4
    down vote













    To diminish gravity, you have to diminish mass. Currently the only way we know how to do that is to convert it explosively into energy. I don't think that will work for you here because if you converted enough of Earth that we would notice the lower gravity, the created energy would probably melt the crust and everyone on it. That makes for a short and tragic story.



    What you probably need here is a dimensional doorway, but that is way beyond our hard science, so the rest of this answer will be in violation of your "science-based" tag.



    If a dimensional doorway existed at the core of our planet, and if a steady stream of core material poured through that doorway and into another plane of existence, and if no reciprocal mass or energy came back through the doorway into our reality, then the planet would slowly (or not so slowly) loose mass. As this happened, the gravity would diminish, but so would the rotational velocity (making our days longer) and the magnetic field (making our days a lot more interesting, i.e. nuclear sunburn). It would get you what you are looking for in terms of failing gravity but the side effects might pose more urgent threats than the sudden weight loss.






    share|improve this answer






















    • Interestingly, covering mass to energy would not reduce its gravitational pull, as mass and energy are equivalent. They are actually represented by the same term in the Einstein field equations.
      – Joel Keene
      Aug 10 at 12:08










    • @JoelKeene, Really? That is non-intuitive. So in a super nova, as a wave of energy propagates out in all directions, that energy has mass and therefore exudes gravity (in addition to its concussive force). I didn't think that energy had mass. The more I hang out on this site, the more I realize that Einstein was the moment when scientists and laymen became separate species.
      – Henry Taylor
      Aug 10 at 13:11

















    up vote
    3
    down vote













    Some ideas:



    • Space is warped, and Earth's gravity well intersects with something on the other side of the warp. The space where our planet once sat is becoming a wormhole, and gravity is failing from our perspective as a result. See the illustration below. This idea is my favorite on this list

    • After the largest underground nuclear bomb test in history, or maybe after another exciting and seemingly innocuous experiment at CERN, the small forces keeping atoms together in the earth's crust have ceased to function normally. The earth is losing mass to radiation at an alarming rate. In some parts of the world, people are dying from radiation sickness; in other parts, people are unable to reproduce, and in other parts the radiation exposure is minimal but the gravitational effects are nonetheless observable. Furthermore, this loss of mass results in large cave-ins, and cities falling into the ground like in Centralia PA.

    • Mass hysteria. It's not really happening, just everybody thinks it is.

    • Earth's gravity has been slowly dying for a long time, but the change was imperceptibly small until now. Now the change is accelerating asymptotically. It only seems to be affecting our planet, but really this is just a stage in the lifecycle of every planet, and we haven't observed it before.

    • Beings from another spacial dimension create some 5th dimensional equivalent to a dyson-sphere around our planet.

    • Beings from this spacial dimension create an actual dyson-sphere around our planet

    Hope this helps!






    share|improve this answer
















    • 2




      Mass hysteria is intriguing. It could open some more psychology to the story.
      – SEK1977
      Aug 9 at 20:26

















    up vote
    3
    down vote













    Your best bet is to use gravitons, the quantum particle/fundamental force-carrying particle of gravity. Dark matter is also a good candidate. Gravitons are similar to photons, the quantum particle of the electromagnetic force (also responsible for giving us light). Like Photons, gravitons are thought to have zero rest mass. This means gravitons travel forever at the speed of light and do not decay. However, gravitons could actually have a very small rest mass which would theoretically allow them to decay into other (lighter) particles.



    Here are some options:



    • Gravitons, which have some infinitesimal but nonzero rest mass, are finally starting to decay en-mass into other force-carrying particles such as photons or gluons.


    • Society is emulated within a computer and is failing to floating point arithmetic errors. For instance in Javascript .1 + .2 = 0.30000000000000004. For instance plank's constant could be wrong.


    • Dark Matter/Dark Energy - Physicists today observe large objects being attracted to certain areas of space that appear to be empty. Since we cannot observe what is attracting matter to these places other than that the attraction is happening via the gravitational force, we assume there is some massive body that is invisible to detection attracting everything. This scenario gives you the greatest flexibility since we really don't understand too much about Dark Matter/Energy. You could make it another form of life that exists in alternate dimensions or just have dark matter and energy be moving around the universe causing havoc.






    share|improve this answer



























      up vote
      3
      down vote













      Gravity As A Wave



      Currently we don't know if gravity is a wave or a particle, but I've read it theorized that it's both, similar to photons. If we assume that gravity can exist as a wave, then perhaps a wave that happen to be the inverse of the gravitational wave signature of Earth's gravity well could cancel out that waveform an therefore nullify gravity. I'm not sure if destructive interference would actually apply to gravity waves, but maybe!



      I would expect something like that to be temporary or fleeting, but there are plenty of cosmological events that might create gravity waves. It could even be a failed human experiment or intentional attack from an outside force.



      Destructive Interference






      share|improve this answer
















      • 3




        Welcome to Worldbuilding! Just so you're aware, they have experimentally verified that gravitational waves (i.e. what it looks like you're talking about) do exist. It's a bit more complicated than this but the general idea of putting something that gives an equally strong gravitational field but with the force in the opposite direction would cancel out Earth's gravity at a point/small region. Gravitational waves are just a perturbation and those emitted by Earth are negligibly weak, but if you had a pair of merging black holes relatively nearby, then indeed, who knows what the result would be?
        – Mithrandir24601
        Aug 9 at 23:27

















      up vote
      2
      down vote













      This wouldn't necessarily reduce gravity, but it would reduce the sensation of gravity.



      Make the Earth start Spinning faster for some reason.



      Maybe the core's Dynamo got stronger due to (insert handwave here) resulting in the Earth spinning faster due to the Handwave Principle of Handaway.



      The centrifugal force would mean that everything on Earth would feel a slight force push them upward, and the net force felt by things on Earth would be weaker.






      share|improve this answer




















      • This would only affect gravity perpendicular to the axis of rotation. Earth already has different axial and equatorial diameters, not to mention gravities. By the time you get enough spin to counteract gravity entirely, you'd have a lot of shape deformation, not to mention some pretty crazy gravity differences as you change latitude
        – Punintended
        Aug 9 at 19:36







      • 2




        @Punintended If you get enough spin to counteract gravity, that would necessarily rip the Earth apart. Yes, increasing the Earth's spin would not result in equal gravity, but it would mean that you might get crazy weather and ocean currents, making the consequences even worse, which is good for the story.
        – Totillity
        Aug 9 at 19:40










      • The planet Mesklin from the old sci-fi classic "Mission of Gravity" was a high-g world spinning fast enough that the gravity was human-survivable (although not comfortable) at the equator.
        – Tim B♦
        Aug 9 at 20:59

















      up vote
      2
      down vote













      While you can't turn off gravity without getting rid of mass, it's possible to mess it up by counteracting it with another invisible force pointing in a different direction. For instance, when you're in a car that takes a sharp turn, you can feel being pushed outward. This is the centrifugal force, and it can even dominate over gravity when the turn is sharp enough, in which case the car flips over.



      How is this relevant to your case? See, we all experience a centrifugal force because of the earth's spin. A fraction of this force counteracts gravity (which always points to the core), however the effect is tiny because the earth spins slowly and has a huge radius: The closer you are at the equator, the stronger the effect. So, if you want to mess up gravity on earth, you can do so by making it spin faster.



      There's actually a formula you can use to calculate the centrifugal force, and compare it to gravity. Without getting into details, you can do some back-of-the-envelope calculations to determine that earth's gravity would start getting messed up by centrifugal forces if it were to spin ~10 times faster. That is, if the duration of a day (sunrise to sunset) were ~1hr long.



      How exactly would earth's gravity get messed up? In short, people at the poles would be unaffected, while those at the equator would basically start floating around (along with the entirety of the atmosphere). Things wouldn't be flung into space, though, because of inertia. People in between would feel lighter (reduced gravity), as well as experience a mysterious pull toward the equator (i.e., southward in the northern hemisphere, and northward in the southern hemisphere).



      In the above apocalyptic scenario, you can imagine that, as the earth spins faster and faster, a certain priviledged few would travel hurriedly toward the poles as the rest of humanity (as well as the atmosphere) gets pulled increasingly strongly toward the equator. It would be a very bizarre situation indeed, as what we all perceive as "down" would no longer feel like "down" anymore; as the earth's spins increases, "down" would increasingly be directed toward the horizon and in the direction of the equator. Not to mention you'd feel lighter and lighter, which, under "normal" conditions, is quite a pleasurable experience.



      There's a catch, of course: How do you actually spin the earth up in a fairly reasonable fashion as to make for a dramatic human story that's actually based on well-established scientific principles? One way I could think of, is reducing the earth's moment of inertia; i.e., changing its internal mass distribution. The more concentrated a body's mass is distributed along the rotation axis, the faster it rotates (conservation of angular momentum). So, without changing anything on the surface where the human drama unfolds, you need a mechanism that can plausibly cause the internal mass to get more concentrated toward the core.



      By playing around with some numbers, it turns out that you need to cram ~99% of the earth's mass right at the core to make the earth spin faster by a factor of ~10. The only mechanism I can think of that could possibly cause such a huge re-distribution of mass, is a swarm of black holes orbiting around each other in the vicinity of the earth's core. Note that these would have to be tiny, since an earth-mass black hole is ~1cm in size and you obviously don't want to add any mass to the earth, since that would mess up "natural" gravity.



      In any case, I didn't investigate the dynamics of a swarm of black holes; i.e., I don't know how quickly they could gobble up a good fraction of the earth's mass as to make the spin-up effect noticeable within a reasonable human timeframe (e.g., years of months). I can't even comment on the stability of such a hypothetical system, since I didn't do any pertinent calculations.






      share|improve this answer




















      • Just read an article about what would happen if the earth spun faster popsci.com/earth-spin-faster#page-4 !
        – Jaco Briers
        Aug 10 at 18:00

















      up vote
      1
      down vote













      I would suggest something in the lines of what happened at the end of Melancholia: a second large mass comes very close (and for some reason stays put at the required distance to make the story interesting), pretty much deforming gravity on Earth. If you need it to be transparent, non-spherically shaped, anything works really. Not every piece of debris (or alien planet-ship) is perfectly spherical.



      For more discussion on the plausibility and effects of such an event, see these network questions:



      https://scifi.stackexchange.com/questions/19068/is-the-melancholia-dance-of-death-possible



      https://physics.stackexchange.com/questions/15083/is-melancholias-orbit-impossible



      Note I am suggesting some articifial means to keep the large second object in place, enough to disturb pretty much everything gravity related, without breaking any of the underlying physics.
      Also be aware that normal gravity is the single most important thing keeping our atmosphere together.






      share|improve this answer



























        up vote
        1
        down vote













        Thinking about this, I realised that the shell theorem is your friend, and would need to stretch reality a lot less than some other solutions. I may have to backtrack a bit to explain this, as it is indeed based on hard science.



        The problem in the question is that as far as we know, gravity is basically, positive only - there isn't negative (anti) gravity, or gravity shields. So the classic solutions have to centre around either mass reduction, or increasingly unlikely/pseudoscientific explanations with implausible implications.



        But there is another way gravity can fail. Let's start with the shell theorem. This is a mathematical/physics principle, that if you are anywhere inside a hollow symmetrical spherical shell, of any thickness, you don't experience any net gravity from it - it all cancels out. (Less mass but closer on one side, more mass but further away on the other, and they always exactly match).



        But suppose mass appears above you, that isn't part of a symmetrical shell. Now, you will experience some reduction in earths gravity locally, because of the upward pull. Just not very much.



        You'll have to check this out, but I guess this could work, especially given that the question allows for limited scientific plausibility:



        1. a research satellite is launched to perform some kind of extreme research in the vacuum of space, in some suitable orbit just outside earth's atmosphere.


        2. Unknown to those running it, and for [handwave] reasons, the satellite is acting as some kind of dense matter attractor - maybe it warps space or leaves behind some particle, or locally perturbs the vacuum state.


        3. The end result is that a series of mini-black holes appear in its wake, conveniently growing and of a size sufficient to influence earths surface gravity below.


        4. The satellite of course fails, after one or 2 orbits, but the mass concentrations continue to grow. They don't form a symmetrical shell, but a linear trail. They inherit the velocity of the satellite, so they don't immediately just fall inward to earth?


        5. The earth isn't affected much in its orbit - at least not immediately - but if figures out carefully, maybe there would be a local/regional impact on surface gravity, with scope for it to get worse, and other side effects, without having to "break" physics or cause immediate planetary breakup.....? It would have a more severe effect on high altitude objects, as well, which might be useful in a story.






        share|improve this answer



























          up vote
          1
          down vote













          Negative mass



          • We had taken for granted for ages that gravity is only attractive. But it turned out that it wasn't.


          • While trying to create chambers of absolute vacuum, it turned out that speed of effect of gravitation is related to the absolute amount of gravitational entropy ( how much of the available mass of the universe can be present at a given location).


          • By creating a perfect vacuum scientists were able to stop the effects of gravity within a region.


          • What was originally understood as vacuum wasn't really vacuum, but a region of space of negative mass which in the presence of other positive behaved like vacuum.



          • Immediately new research begun and more and more people start researching on this. Meanwhile it turns out that by creating more and more perfect vacuums, we are creating more regions of negative mass.



            • This negative mass is termed as Anti Dark Matter for it only interacts through gravitational forces and is repulsive.


          • Termination of a vacuum doesn't always ensure that all the positive mass covers up the negative.



          • As the gravitational interactions are very weak it was only noticed many more years later that the experiments which were run had caused a small leakage of negative mass that had somehow made it to the near the earth's outer core where it nicely found and equilibrium and settled.



            • As Earth passes through various regions of space, overtime this negative mass at the core had been collecting more and more of negative mass, and it is growing exponentially.


            • It is cancelling out the effects of the earth's core and mantle by overlapping and creating regions of 'vacuum'







          share|improve this answer



























            up vote
            0
            down vote













            Gravity is caused by bending of space time (IRL).



            A big corporation has worked out a way of flattening it artificially - effectively "antigravity", and sells it as a product.






            share|improve this answer



























              up vote
              0
              down vote













              The changing/failing gravity is a floating point error in the universe simulation.



              Many existing simulations of varying complexity use floating point numbers for their calculations. For example, the Unity game engine uses a floating point number for gravity strength. In this case, the floating point number would be the lifetime or duration of the gravity effect. It could even be tracking the lifetime of the universe, and the gravity is just a side effect.






              share|improve this answer




















              • Welcome to Worldbuilding, Jarod S! If you have a moment, please take the tour and visit the help center to learn more about the site. You may also find Worldbuilding Meta and The Sandbox (both of which require 5 rep to post on) useful. Here is a meta post on the culture and style of Worldbuilding.SE, just to help you understand our scope and methods, and how we do things here. Have fun!
                – Gryphon
                Aug 10 at 17:13

















              up vote
              0
              down vote













              Earth isn't real. I'm borrowing an idea from fiction:



              Specifically, Mutineer's Moon, the first book of David Weber's Fifth Imperium series. They needed to hide a very large starship in our solar system--they took the surface off the Moon and put it on the starship, tossed the core into the sun and took the moon's place.



              Lets take this further--at some time in the past aliens replaced Earth with a hidden starship. The fakery is good enough that even sounding of the innards doesen't reveal that it's starship and not planet. However, the starship is nowhere near as massive as the planet, it had to use a gravity generator to make things look real.



              The maintenance system on the starship is failing. There are actually many gravity generators in the starship so a failure doesn't leave the planet coming apart, but now they are wearing out. As generators fail the others take up the load--but not quite evenly and the additional load on them speeds their own failure and once enough have failed they can't maintain 1g anymore.



              When all the generators are dead there will still be some gravity due to the mass of the ship and the mass of the crust that was laid over the ship, but it won't be anything like 1g and won't keep an atmosphere for the long term.






              share|improve this answer



























                up vote
                0
                down vote













                A great many people have independently considered the idea that gravity doesn't attract, it repels. The attraction observed between neighboring bodies is do to partial shielding of the bodies by one another. The shielding is small and proportional to mass. This doesn't actually work if you dig into the details, but you have to dig pretty deeply. The advantage now is that the source of gravitational pressure can be entirely divorced from mass and located a very long way away. Enabling this view of gravity would open up all kinds of options. It would seem that you are on the cusp of some event horizon that is about to isolate our corner of the universe from it's source of gravitination.






                share|improve this answer




















                • Do you have anything to back up your statement that gravity is repulsive? Because gravitational attraction between bodies has been measured in a lab
                  – L.Dutch♦
                  Aug 11 at 19:49










                • @L.Dutch I said it doesn't work. I know it doesn't work. But it is far from obvious that it doesn't work, and it opens up a lot of options. Hence "enabling this view ..."
                  – Phil Sweet
                  Aug 12 at 16:05

















                up vote
                0
                down vote













                Alien Transportation System



                OK, this one sounds a bit "space opera"-ish, but you can probably dress it up and make it pretty.



                Aliens from an alternate dimension/universe/what-have-you use "our" dimension/universe/etc as a convenient way to travel great distances in "their" dimension, because the mapping between the dimensions is such that traveling (a short distance) in "our" dimension is equivalent to traveling (a very very long distance) in theirs. Jumping from a "small" dimension (theirs) to a "big" dimension (ours) requires very little energy - but traveling from a "big" dimension to a "small" dimension requires A Lot of energy - and the energy (and thus the mass) is removed from the local area near the jump site and transferred to the "small" dimension. And, again unfortunately, the Sol system just happens to be conveniently located for their use - so they're sucking the energy/mass out of our area. Oh, BTW, that means the sun is going to go nova. Sorry about your luck, humans - BUT as a consolation prize the aliens have offered each and every human a set of genuine Alien Ginsu Steak Knives! They slice! They dice! They make your star blow up! Wooooo!






                share|improve this answer



























                  up vote
                  -1
                  down vote













                  No



                  Gravity doesn't "fail", any more than the other fundamental forces fail. If they could, we would live in an entirely different universe. If any of the forces "started to fail", so many catastrophic things would happen all at once that there would be no grace period.






                  share|improve this answer
















                  • 3




                    A brief look at the history of science yields that long-held theories about the way the universe works have a broad tendency toward failure. We're constantly revising our understanding of the universe because we're frequently wrong. Gravity is just a theory which we hold to be true because we've never observed it failing, but without a solid understanding of how much probabilistic weight a single instance of nonfailure yields, our lives are too short to develop the kind of certainty this answer reflects; at least not without some higher source of absolute knowledge.
                    – boxcartenant
                    Aug 9 at 19:31







                  • 1




                    I'm with @boxcartenant. I was regularly frustrated by a young woman taking nursing classes because she would spout today's medical understanding and complete her statement with the phrase, "and that's a medical fact!" She's 20 something. I'm 50 something. I've lived long enough to know some of the so-called medical facts she's spouting didn't even exist at one point or changed to accomodate recent research. Our understanding of just about everything is hardly complete. To paraphrase the movie Volcano, "we're scientists, 'certainty' is a big word."
                    – JBH
                    Aug 9 at 19:51






                  • 1




                    @boxcartenant: Long-held theories are being revised, not invalidated. Newton's Law of Gravity still works for most purposes. There have been some big changes, but they tend to explain new things. Photons explained the previously unexplained cases of the photoelectric effect and black-body radiation, but optics wasn't really affected. We know that physical laws are pretty much the same all over the observable Universe.
                    – David Thornley
                    Aug 9 at 20:18






                  • 1




                    @DavidThornley Good feedback. I think, though, a revision is essentially a partial invalidation. We'll never completely invalidate gravity, because we can plainly see that things are falling, but we might revise our understanding of it so significantly that it would be unrecognizable against its current form; so while the laws might be the same everywhere, their application may be entirely different from what we currently suppose.
                    – boxcartenant
                    Aug 9 at 20:38






                  • 1




                    This is not a reality check.
                    – Theraot
                    Aug 10 at 1:23










                  Your Answer




                  StackExchange.ifUsing("editor", function ()
                  return StackExchange.using("mathjaxEditing", function ()
                  StackExchange.MarkdownEditor.creationCallbacks.add(function (editor, postfix)
                  StackExchange.mathjaxEditing.prepareWmdForMathJax(editor, postfix, [["$", "$"], ["\\(","\\)"]]);
                  );
                  );
                  , "mathjax-editing");

                  StackExchange.ready(function()
                  var channelOptions =
                  tags: "".split(" "),
                  id: "579"
                  ;
                  initTagRenderer("".split(" "), "".split(" "), channelOptions);

                  StackExchange.using("externalEditor", function()
                  // Have to fire editor after snippets, if snippets enabled
                  if (StackExchange.settings.snippets.snippetsEnabled)
                  StackExchange.using("snippets", function()
                  createEditor();
                  );

                  else
                  createEditor();

                  );

                  function createEditor()
                  StackExchange.prepareEditor(
                  heartbeatType: 'answer',
                  convertImagesToLinks: false,
                  noModals: false,
                  showLowRepImageUploadWarning: true,
                  reputationToPostImages: null,
                  bindNavPrevention: true,
                  postfix: "",
                  noCode: true, onDemand: true,
                  discardSelector: ".discard-answer"
                  ,immediatelyShowMarkdownHelp:true
                  );



                  );













                   

                  draft saved


                  draft discarded


















                  StackExchange.ready(
                  function ()
                  StackExchange.openid.initPostLogin('.new-post-login', 'https%3a%2f%2fworldbuilding.stackexchange.com%2fquestions%2f120899%2fplausible-hypothesis-on-why-gravity-might-fail-on-earth%23new-answer', 'question_page');

                  );

                  Post as a guest






























                  22 Answers
                  22






                  active

                  oldest

                  votes








                  22 Answers
                  22






                  active

                  oldest

                  votes









                  active

                  oldest

                  votes






                  active

                  oldest

                  votes








                  up vote
                  3
                  down vote



                  accepted










                  Dark Energy / The Big Rip



                  There is a possibility that this will eventually happen. It depends on the energy density of dark energy in the universe, which we are currently unable to measure accurately enough to tell.



                  Strictly speaking, this is not gravity failing so much as the expansion of space accelerating, but for all intents and purposes the results are the same. Basically everything in the universe is accelerating away from everything else, even on tiny scales, but the rate of expansion is greatly overpowered by gravity except on intergalactic scales. But if the expansion continues to accelerate, eventually it will overpower gravity.



                  This will actually create a predictable sequence of events that can happen on human-measurable scales. It could make a great setting for a frantic race against the clock as scientists struggle to figure out a way of escaping the universe before everything ends.



                  Using one hypothetical example using real equations, the Big Rip could happen 22 billion years from now. (This will be long after our own solar system is gone, but humanity's descendants could still be around).



                  60 million years before the end, galaxies become gravitationally unbound. Your setting is a lone solar system in a starless void. It is still habitable but the knowledge that there was once such a thing as stars could be long forgotten.



                  About 3 months before the end, the gravitational attraction between the planet and its star is overpowered by the expansion of space. The planet begins to move away from its star, first slowly and then picking up speed until people are forced to use technology to survive the cold. But things are going to get much worse...



                  Gravity starts to weaken. Quakes and cataclysmic upheaval become commonplace as the pressure holding the planet's insides inside diminishes. The atmosphere grows thin. The oceans boil. In the last few minutes before the end, the gravitational force cannot hold the planet together anymore and every planet in the solar system explodes at around the same time that the distant sun evaporates into the void.



                  Anyone who manages to survive in spacecrafts have only a few minutes left before the accelerating expansion of space overpowers the binding energy of matter itself. There might be a few moments where they can actually feel it building up inside them before it rips apart their bodies, then their molecules, atoms, and finally subatomic particles themselves.



                  You can adjust the timing if you want to introduce any kind of handwavium physical laws that we don't know about yet.






                  share|improve this answer
























                    up vote
                    3
                    down vote



                    accepted










                    Dark Energy / The Big Rip



                    There is a possibility that this will eventually happen. It depends on the energy density of dark energy in the universe, which we are currently unable to measure accurately enough to tell.



                    Strictly speaking, this is not gravity failing so much as the expansion of space accelerating, but for all intents and purposes the results are the same. Basically everything in the universe is accelerating away from everything else, even on tiny scales, but the rate of expansion is greatly overpowered by gravity except on intergalactic scales. But if the expansion continues to accelerate, eventually it will overpower gravity.



                    This will actually create a predictable sequence of events that can happen on human-measurable scales. It could make a great setting for a frantic race against the clock as scientists struggle to figure out a way of escaping the universe before everything ends.



                    Using one hypothetical example using real equations, the Big Rip could happen 22 billion years from now. (This will be long after our own solar system is gone, but humanity's descendants could still be around).



                    60 million years before the end, galaxies become gravitationally unbound. Your setting is a lone solar system in a starless void. It is still habitable but the knowledge that there was once such a thing as stars could be long forgotten.



                    About 3 months before the end, the gravitational attraction between the planet and its star is overpowered by the expansion of space. The planet begins to move away from its star, first slowly and then picking up speed until people are forced to use technology to survive the cold. But things are going to get much worse...



                    Gravity starts to weaken. Quakes and cataclysmic upheaval become commonplace as the pressure holding the planet's insides inside diminishes. The atmosphere grows thin. The oceans boil. In the last few minutes before the end, the gravitational force cannot hold the planet together anymore and every planet in the solar system explodes at around the same time that the distant sun evaporates into the void.



                    Anyone who manages to survive in spacecrafts have only a few minutes left before the accelerating expansion of space overpowers the binding energy of matter itself. There might be a few moments where they can actually feel it building up inside them before it rips apart their bodies, then their molecules, atoms, and finally subatomic particles themselves.



                    You can adjust the timing if you want to introduce any kind of handwavium physical laws that we don't know about yet.






                    share|improve this answer






















                      up vote
                      3
                      down vote



                      accepted







                      up vote
                      3
                      down vote



                      accepted






                      Dark Energy / The Big Rip



                      There is a possibility that this will eventually happen. It depends on the energy density of dark energy in the universe, which we are currently unable to measure accurately enough to tell.



                      Strictly speaking, this is not gravity failing so much as the expansion of space accelerating, but for all intents and purposes the results are the same. Basically everything in the universe is accelerating away from everything else, even on tiny scales, but the rate of expansion is greatly overpowered by gravity except on intergalactic scales. But if the expansion continues to accelerate, eventually it will overpower gravity.



                      This will actually create a predictable sequence of events that can happen on human-measurable scales. It could make a great setting for a frantic race against the clock as scientists struggle to figure out a way of escaping the universe before everything ends.



                      Using one hypothetical example using real equations, the Big Rip could happen 22 billion years from now. (This will be long after our own solar system is gone, but humanity's descendants could still be around).



                      60 million years before the end, galaxies become gravitationally unbound. Your setting is a lone solar system in a starless void. It is still habitable but the knowledge that there was once such a thing as stars could be long forgotten.



                      About 3 months before the end, the gravitational attraction between the planet and its star is overpowered by the expansion of space. The planet begins to move away from its star, first slowly and then picking up speed until people are forced to use technology to survive the cold. But things are going to get much worse...



                      Gravity starts to weaken. Quakes and cataclysmic upheaval become commonplace as the pressure holding the planet's insides inside diminishes. The atmosphere grows thin. The oceans boil. In the last few minutes before the end, the gravitational force cannot hold the planet together anymore and every planet in the solar system explodes at around the same time that the distant sun evaporates into the void.



                      Anyone who manages to survive in spacecrafts have only a few minutes left before the accelerating expansion of space overpowers the binding energy of matter itself. There might be a few moments where they can actually feel it building up inside them before it rips apart their bodies, then their molecules, atoms, and finally subatomic particles themselves.



                      You can adjust the timing if you want to introduce any kind of handwavium physical laws that we don't know about yet.






                      share|improve this answer












                      Dark Energy / The Big Rip



                      There is a possibility that this will eventually happen. It depends on the energy density of dark energy in the universe, which we are currently unable to measure accurately enough to tell.



                      Strictly speaking, this is not gravity failing so much as the expansion of space accelerating, but for all intents and purposes the results are the same. Basically everything in the universe is accelerating away from everything else, even on tiny scales, but the rate of expansion is greatly overpowered by gravity except on intergalactic scales. But if the expansion continues to accelerate, eventually it will overpower gravity.



                      This will actually create a predictable sequence of events that can happen on human-measurable scales. It could make a great setting for a frantic race against the clock as scientists struggle to figure out a way of escaping the universe before everything ends.



                      Using one hypothetical example using real equations, the Big Rip could happen 22 billion years from now. (This will be long after our own solar system is gone, but humanity's descendants could still be around).



                      60 million years before the end, galaxies become gravitationally unbound. Your setting is a lone solar system in a starless void. It is still habitable but the knowledge that there was once such a thing as stars could be long forgotten.



                      About 3 months before the end, the gravitational attraction between the planet and its star is overpowered by the expansion of space. The planet begins to move away from its star, first slowly and then picking up speed until people are forced to use technology to survive the cold. But things are going to get much worse...



                      Gravity starts to weaken. Quakes and cataclysmic upheaval become commonplace as the pressure holding the planet's insides inside diminishes. The atmosphere grows thin. The oceans boil. In the last few minutes before the end, the gravitational force cannot hold the planet together anymore and every planet in the solar system explodes at around the same time that the distant sun evaporates into the void.



                      Anyone who manages to survive in spacecrafts have only a few minutes left before the accelerating expansion of space overpowers the binding energy of matter itself. There might be a few moments where they can actually feel it building up inside them before it rips apart their bodies, then their molecules, atoms, and finally subatomic particles themselves.



                      You can adjust the timing if you want to introduce any kind of handwavium physical laws that we don't know about yet.







                      share|improve this answer












                      share|improve this answer



                      share|improve this answer










                      answered Aug 12 at 12:46









                      IndigoFenix

                      13.4k12359




                      13.4k12359




















                          up vote
                          39
                          down vote













                          Totally different idea.



                          In late 2283, a research team at the University of New Tokyo finally cracked the Unified Field problem. This gained them one of the most prestigious Nobel Prizes ever awarded in history.



                          The results of this were initially amazing: it allowed on demand gravity control, the benefits of which were immense. Needless to say, everyone started making gravity control devices for vehicle levitation, space exploration, construction, etc. You name it, we found an application for gravity control.



                          However, what nobody had anticipated was that the count of particles moderating the forces in the unified field theorem had been a balanced set of parameters, and the changes that occurred as a result of gravity control led to an ever increasing instability.



                          In essence we'd been living in a local minimum for this parameter equation where everything was stable, but gravity control eventually pushed us out of this local minimum into an incredibly unstable landscape. One of the major side effects of this was that gravity slowly, but irreversibly, started to decrease.






                          share|improve this answer
















                          • 21




                            This almost sounds like an allegory for the Earth's dependence on oil. We become dependent on a specific resource and start to use it for everything everywhere. That same widespread deployment turns around and bites us. Good suggestion.
                            – Ian Johnson
                            Aug 10 at 3:02






                          • 5




                            I was just reading this answer thinking how did I not hear about this... then I reread the date!
                            – Liath
                            Aug 10 at 12:28










                          • I like this answer because it provides your story with so many possibilities, the hubris and arrogance, and ignorance, of the society that has become dependent on technology. Possibly also an answer too to the Fermi Paradox :) A great filter indeed.
                            – flox
                            Aug 12 at 17:26














                          up vote
                          39
                          down vote













                          Totally different idea.



                          In late 2283, a research team at the University of New Tokyo finally cracked the Unified Field problem. This gained them one of the most prestigious Nobel Prizes ever awarded in history.



                          The results of this were initially amazing: it allowed on demand gravity control, the benefits of which were immense. Needless to say, everyone started making gravity control devices for vehicle levitation, space exploration, construction, etc. You name it, we found an application for gravity control.



                          However, what nobody had anticipated was that the count of particles moderating the forces in the unified field theorem had been a balanced set of parameters, and the changes that occurred as a result of gravity control led to an ever increasing instability.



                          In essence we'd been living in a local minimum for this parameter equation where everything was stable, but gravity control eventually pushed us out of this local minimum into an incredibly unstable landscape. One of the major side effects of this was that gravity slowly, but irreversibly, started to decrease.






                          share|improve this answer
















                          • 21




                            This almost sounds like an allegory for the Earth's dependence on oil. We become dependent on a specific resource and start to use it for everything everywhere. That same widespread deployment turns around and bites us. Good suggestion.
                            – Ian Johnson
                            Aug 10 at 3:02






                          • 5




                            I was just reading this answer thinking how did I not hear about this... then I reread the date!
                            – Liath
                            Aug 10 at 12:28










                          • I like this answer because it provides your story with so many possibilities, the hubris and arrogance, and ignorance, of the society that has become dependent on technology. Possibly also an answer too to the Fermi Paradox :) A great filter indeed.
                            – flox
                            Aug 12 at 17:26












                          up vote
                          39
                          down vote










                          up vote
                          39
                          down vote









                          Totally different idea.



                          In late 2283, a research team at the University of New Tokyo finally cracked the Unified Field problem. This gained them one of the most prestigious Nobel Prizes ever awarded in history.



                          The results of this were initially amazing: it allowed on demand gravity control, the benefits of which were immense. Needless to say, everyone started making gravity control devices for vehicle levitation, space exploration, construction, etc. You name it, we found an application for gravity control.



                          However, what nobody had anticipated was that the count of particles moderating the forces in the unified field theorem had been a balanced set of parameters, and the changes that occurred as a result of gravity control led to an ever increasing instability.



                          In essence we'd been living in a local minimum for this parameter equation where everything was stable, but gravity control eventually pushed us out of this local minimum into an incredibly unstable landscape. One of the major side effects of this was that gravity slowly, but irreversibly, started to decrease.






                          share|improve this answer












                          Totally different idea.



                          In late 2283, a research team at the University of New Tokyo finally cracked the Unified Field problem. This gained them one of the most prestigious Nobel Prizes ever awarded in history.



                          The results of this were initially amazing: it allowed on demand gravity control, the benefits of which were immense. Needless to say, everyone started making gravity control devices for vehicle levitation, space exploration, construction, etc. You name it, we found an application for gravity control.



                          However, what nobody had anticipated was that the count of particles moderating the forces in the unified field theorem had been a balanced set of parameters, and the changes that occurred as a result of gravity control led to an ever increasing instability.



                          In essence we'd been living in a local minimum for this parameter equation where everything was stable, but gravity control eventually pushed us out of this local minimum into an incredibly unstable landscape. One of the major side effects of this was that gravity slowly, but irreversibly, started to decrease.







                          share|improve this answer












                          share|improve this answer



                          share|improve this answer










                          answered Aug 10 at 1:24









                          dgnuff

                          41114




                          41114







                          • 21




                            This almost sounds like an allegory for the Earth's dependence on oil. We become dependent on a specific resource and start to use it for everything everywhere. That same widespread deployment turns around and bites us. Good suggestion.
                            – Ian Johnson
                            Aug 10 at 3:02






                          • 5




                            I was just reading this answer thinking how did I not hear about this... then I reread the date!
                            – Liath
                            Aug 10 at 12:28










                          • I like this answer because it provides your story with so many possibilities, the hubris and arrogance, and ignorance, of the society that has become dependent on technology. Possibly also an answer too to the Fermi Paradox :) A great filter indeed.
                            – flox
                            Aug 12 at 17:26












                          • 21




                            This almost sounds like an allegory for the Earth's dependence on oil. We become dependent on a specific resource and start to use it for everything everywhere. That same widespread deployment turns around and bites us. Good suggestion.
                            – Ian Johnson
                            Aug 10 at 3:02






                          • 5




                            I was just reading this answer thinking how did I not hear about this... then I reread the date!
                            – Liath
                            Aug 10 at 12:28










                          • I like this answer because it provides your story with so many possibilities, the hubris and arrogance, and ignorance, of the society that has become dependent on technology. Possibly also an answer too to the Fermi Paradox :) A great filter indeed.
                            – flox
                            Aug 12 at 17:26







                          21




                          21




                          This almost sounds like an allegory for the Earth's dependence on oil. We become dependent on a specific resource and start to use it for everything everywhere. That same widespread deployment turns around and bites us. Good suggestion.
                          – Ian Johnson
                          Aug 10 at 3:02




                          This almost sounds like an allegory for the Earth's dependence on oil. We become dependent on a specific resource and start to use it for everything everywhere. That same widespread deployment turns around and bites us. Good suggestion.
                          – Ian Johnson
                          Aug 10 at 3:02




                          5




                          5




                          I was just reading this answer thinking how did I not hear about this... then I reread the date!
                          – Liath
                          Aug 10 at 12:28




                          I was just reading this answer thinking how did I not hear about this... then I reread the date!
                          – Liath
                          Aug 10 at 12:28












                          I like this answer because it provides your story with so many possibilities, the hubris and arrogance, and ignorance, of the society that has become dependent on technology. Possibly also an answer too to the Fermi Paradox :) A great filter indeed.
                          – flox
                          Aug 12 at 17:26




                          I like this answer because it provides your story with so many possibilities, the hubris and arrogance, and ignorance, of the society that has become dependent on technology. Possibly also an answer too to the Fermi Paradox :) A great filter indeed.
                          – flox
                          Aug 12 at 17:26










                          up vote
                          25
                          down vote













                          We need to have more fun with this question



                          Keep in mind that gravity is but one characteristic of mass. You can't change just gravity without side effects. Those side effects are valuable to you as they can be used to advance your plot.



                          • A wormhole appears at the center of the planet, pulling mass away from the core. Thanks to the shell theorem, gravity must reduce to zero as you approach the center of the planetary sphere. Thus, the appearance wouldn't be immediately noticable, and if the mass were drawn off slowly enough, it wouldn't be appreciably obvious. Note that there would be side effects, such as liquid core currents changing, which would change the magnetosphere and possibly plate tectonics.


                          • Something in the universe, some ribbon of effect we know nothing about, is passing through our solar system and, thus, our planet is passing through it. This ribbon is changing the positive charge strength forcing atomic nucleai to spread apart. Not only is this reducing the density of planetary mass (and therefore gravity), but it's also causing the planet to increase in circumference.


                          • Something unseen is passing through the solar system. A "dark planet" (or, perhaps of greater interest, a ribbon of dark asteroids) with sufficient mass as any point of its passing to change the effective gravity on Earth. The planet or ribbon can pass through on a parabolic arc such that it would affect the Earth for most of a year. The side effect is that Earthlings standing on the side closest to the dark planet/belt/ribbon would be lighter, while those on the opposite side would feel heavier. Earthquakes would be normal. Shifting the Earth's orbit (perhaps moving it further from the sun, thereby solving global warming) would be expected.


                          • Q is really bored and decides to change the gravitational constant of the universe — but only in the Sol system. Not only would this be really hard to figure out what was going on, but ships leaving the system would sail through a breakwater (I'm using the wrong word, that point in the ocean when leaving a beach when the undersea landmass drops away and you get serious wave action. It killed the best friend of Moana's father. It was really sad and should convince all to live in Iowa).


                          • The theory that you can treat gravity as coming from a point source at the center of a sphere is only true when the sphere is reasonably solid or homogenous. Let's have some reaction (let's blame it on the Somalis this time, they built a planetary core implosion device) that caused the inner 25% of the planet's volume to suddenly compress such that a massive void appeared between the new basically neutronium core and the rest of the core/mantle. Not only would this change gravity by pulling a lot of mass further away from the surface, but it would have the side effect of the planet slowly starting to wobble in its rotation until that big marble comes in contact with the rest of the core and begins acting like that child's tooth in a paint can... or until the sudden vacuum causes the planet to violently shrink.


                          What else, my fellow WorldBuildians! In what other ways can be bend, contort, graffiti, or otherwise ignore the fundamental rules of orbital mechanics and classical physics to bring about, if nothing else, the appearance of a lessening gravity?






                          share|improve this answer
















                          • 3




                            You forgot to pay the gravity bill... goo.gl/images/h8NWJN
                            – Jim Garrison
                            Aug 10 at 5:12











                          • This sounds like the most plausible (or, least implausible) of the existing answers as I write this, according to current science.
                            – David Z
                            Aug 10 at 5:45










                          • Is the term you’re looking for “shelf break” or “continental slope?” (source)
                            – J F
                            Aug 10 at 10:57










                          • +1 shell theory
                            – PV22
                            Aug 11 at 17:47














                          up vote
                          25
                          down vote













                          We need to have more fun with this question



                          Keep in mind that gravity is but one characteristic of mass. You can't change just gravity without side effects. Those side effects are valuable to you as they can be used to advance your plot.



                          • A wormhole appears at the center of the planet, pulling mass away from the core. Thanks to the shell theorem, gravity must reduce to zero as you approach the center of the planetary sphere. Thus, the appearance wouldn't be immediately noticable, and if the mass were drawn off slowly enough, it wouldn't be appreciably obvious. Note that there would be side effects, such as liquid core currents changing, which would change the magnetosphere and possibly plate tectonics.


                          • Something in the universe, some ribbon of effect we know nothing about, is passing through our solar system and, thus, our planet is passing through it. This ribbon is changing the positive charge strength forcing atomic nucleai to spread apart. Not only is this reducing the density of planetary mass (and therefore gravity), but it's also causing the planet to increase in circumference.


                          • Something unseen is passing through the solar system. A "dark planet" (or, perhaps of greater interest, a ribbon of dark asteroids) with sufficient mass as any point of its passing to change the effective gravity on Earth. The planet or ribbon can pass through on a parabolic arc such that it would affect the Earth for most of a year. The side effect is that Earthlings standing on the side closest to the dark planet/belt/ribbon would be lighter, while those on the opposite side would feel heavier. Earthquakes would be normal. Shifting the Earth's orbit (perhaps moving it further from the sun, thereby solving global warming) would be expected.


                          • Q is really bored and decides to change the gravitational constant of the universe — but only in the Sol system. Not only would this be really hard to figure out what was going on, but ships leaving the system would sail through a breakwater (I'm using the wrong word, that point in the ocean when leaving a beach when the undersea landmass drops away and you get serious wave action. It killed the best friend of Moana's father. It was really sad and should convince all to live in Iowa).


                          • The theory that you can treat gravity as coming from a point source at the center of a sphere is only true when the sphere is reasonably solid or homogenous. Let's have some reaction (let's blame it on the Somalis this time, they built a planetary core implosion device) that caused the inner 25% of the planet's volume to suddenly compress such that a massive void appeared between the new basically neutronium core and the rest of the core/mantle. Not only would this change gravity by pulling a lot of mass further away from the surface, but it would have the side effect of the planet slowly starting to wobble in its rotation until that big marble comes in contact with the rest of the core and begins acting like that child's tooth in a paint can... or until the sudden vacuum causes the planet to violently shrink.


                          What else, my fellow WorldBuildians! In what other ways can be bend, contort, graffiti, or otherwise ignore the fundamental rules of orbital mechanics and classical physics to bring about, if nothing else, the appearance of a lessening gravity?






                          share|improve this answer
















                          • 3




                            You forgot to pay the gravity bill... goo.gl/images/h8NWJN
                            – Jim Garrison
                            Aug 10 at 5:12











                          • This sounds like the most plausible (or, least implausible) of the existing answers as I write this, according to current science.
                            – David Z
                            Aug 10 at 5:45










                          • Is the term you’re looking for “shelf break” or “continental slope?” (source)
                            – J F
                            Aug 10 at 10:57










                          • +1 shell theory
                            – PV22
                            Aug 11 at 17:47












                          up vote
                          25
                          down vote










                          up vote
                          25
                          down vote









                          We need to have more fun with this question



                          Keep in mind that gravity is but one characteristic of mass. You can't change just gravity without side effects. Those side effects are valuable to you as they can be used to advance your plot.



                          • A wormhole appears at the center of the planet, pulling mass away from the core. Thanks to the shell theorem, gravity must reduce to zero as you approach the center of the planetary sphere. Thus, the appearance wouldn't be immediately noticable, and if the mass were drawn off slowly enough, it wouldn't be appreciably obvious. Note that there would be side effects, such as liquid core currents changing, which would change the magnetosphere and possibly plate tectonics.


                          • Something in the universe, some ribbon of effect we know nothing about, is passing through our solar system and, thus, our planet is passing through it. This ribbon is changing the positive charge strength forcing atomic nucleai to spread apart. Not only is this reducing the density of planetary mass (and therefore gravity), but it's also causing the planet to increase in circumference.


                          • Something unseen is passing through the solar system. A "dark planet" (or, perhaps of greater interest, a ribbon of dark asteroids) with sufficient mass as any point of its passing to change the effective gravity on Earth. The planet or ribbon can pass through on a parabolic arc such that it would affect the Earth for most of a year. The side effect is that Earthlings standing on the side closest to the dark planet/belt/ribbon would be lighter, while those on the opposite side would feel heavier. Earthquakes would be normal. Shifting the Earth's orbit (perhaps moving it further from the sun, thereby solving global warming) would be expected.


                          • Q is really bored and decides to change the gravitational constant of the universe — but only in the Sol system. Not only would this be really hard to figure out what was going on, but ships leaving the system would sail through a breakwater (I'm using the wrong word, that point in the ocean when leaving a beach when the undersea landmass drops away and you get serious wave action. It killed the best friend of Moana's father. It was really sad and should convince all to live in Iowa).


                          • The theory that you can treat gravity as coming from a point source at the center of a sphere is only true when the sphere is reasonably solid or homogenous. Let's have some reaction (let's blame it on the Somalis this time, they built a planetary core implosion device) that caused the inner 25% of the planet's volume to suddenly compress such that a massive void appeared between the new basically neutronium core and the rest of the core/mantle. Not only would this change gravity by pulling a lot of mass further away from the surface, but it would have the side effect of the planet slowly starting to wobble in its rotation until that big marble comes in contact with the rest of the core and begins acting like that child's tooth in a paint can... or until the sudden vacuum causes the planet to violently shrink.


                          What else, my fellow WorldBuildians! In what other ways can be bend, contort, graffiti, or otherwise ignore the fundamental rules of orbital mechanics and classical physics to bring about, if nothing else, the appearance of a lessening gravity?






                          share|improve this answer












                          We need to have more fun with this question



                          Keep in mind that gravity is but one characteristic of mass. You can't change just gravity without side effects. Those side effects are valuable to you as they can be used to advance your plot.



                          • A wormhole appears at the center of the planet, pulling mass away from the core. Thanks to the shell theorem, gravity must reduce to zero as you approach the center of the planetary sphere. Thus, the appearance wouldn't be immediately noticable, and if the mass were drawn off slowly enough, it wouldn't be appreciably obvious. Note that there would be side effects, such as liquid core currents changing, which would change the magnetosphere and possibly plate tectonics.


                          • Something in the universe, some ribbon of effect we know nothing about, is passing through our solar system and, thus, our planet is passing through it. This ribbon is changing the positive charge strength forcing atomic nucleai to spread apart. Not only is this reducing the density of planetary mass (and therefore gravity), but it's also causing the planet to increase in circumference.


                          • Something unseen is passing through the solar system. A "dark planet" (or, perhaps of greater interest, a ribbon of dark asteroids) with sufficient mass as any point of its passing to change the effective gravity on Earth. The planet or ribbon can pass through on a parabolic arc such that it would affect the Earth for most of a year. The side effect is that Earthlings standing on the side closest to the dark planet/belt/ribbon would be lighter, while those on the opposite side would feel heavier. Earthquakes would be normal. Shifting the Earth's orbit (perhaps moving it further from the sun, thereby solving global warming) would be expected.


                          • Q is really bored and decides to change the gravitational constant of the universe — but only in the Sol system. Not only would this be really hard to figure out what was going on, but ships leaving the system would sail through a breakwater (I'm using the wrong word, that point in the ocean when leaving a beach when the undersea landmass drops away and you get serious wave action. It killed the best friend of Moana's father. It was really sad and should convince all to live in Iowa).


                          • The theory that you can treat gravity as coming from a point source at the center of a sphere is only true when the sphere is reasonably solid or homogenous. Let's have some reaction (let's blame it on the Somalis this time, they built a planetary core implosion device) that caused the inner 25% of the planet's volume to suddenly compress such that a massive void appeared between the new basically neutronium core and the rest of the core/mantle. Not only would this change gravity by pulling a lot of mass further away from the surface, but it would have the side effect of the planet slowly starting to wobble in its rotation until that big marble comes in contact with the rest of the core and begins acting like that child's tooth in a paint can... or until the sudden vacuum causes the planet to violently shrink.


                          What else, my fellow WorldBuildians! In what other ways can be bend, contort, graffiti, or otherwise ignore the fundamental rules of orbital mechanics and classical physics to bring about, if nothing else, the appearance of a lessening gravity?







                          share|improve this answer












                          share|improve this answer



                          share|improve this answer










                          answered Aug 9 at 19:43









                          JBH

                          32.2k578153




                          32.2k578153







                          • 3




                            You forgot to pay the gravity bill... goo.gl/images/h8NWJN
                            – Jim Garrison
                            Aug 10 at 5:12











                          • This sounds like the most plausible (or, least implausible) of the existing answers as I write this, according to current science.
                            – David Z
                            Aug 10 at 5:45










                          • Is the term you’re looking for “shelf break” or “continental slope?” (source)
                            – J F
                            Aug 10 at 10:57










                          • +1 shell theory
                            – PV22
                            Aug 11 at 17:47












                          • 3




                            You forgot to pay the gravity bill... goo.gl/images/h8NWJN
                            – Jim Garrison
                            Aug 10 at 5:12











                          • This sounds like the most plausible (or, least implausible) of the existing answers as I write this, according to current science.
                            – David Z
                            Aug 10 at 5:45










                          • Is the term you’re looking for “shelf break” or “continental slope?” (source)
                            – J F
                            Aug 10 at 10:57










                          • +1 shell theory
                            – PV22
                            Aug 11 at 17:47







                          3




                          3




                          You forgot to pay the gravity bill... goo.gl/images/h8NWJN
                          – Jim Garrison
                          Aug 10 at 5:12





                          You forgot to pay the gravity bill... goo.gl/images/h8NWJN
                          – Jim Garrison
                          Aug 10 at 5:12













                          This sounds like the most plausible (or, least implausible) of the existing answers as I write this, according to current science.
                          – David Z
                          Aug 10 at 5:45




                          This sounds like the most plausible (or, least implausible) of the existing answers as I write this, according to current science.
                          – David Z
                          Aug 10 at 5:45












                          Is the term you’re looking for “shelf break” or “continental slope?” (source)
                          – J F
                          Aug 10 at 10:57




                          Is the term you’re looking for “shelf break” or “continental slope?” (source)
                          – J F
                          Aug 10 at 10:57












                          +1 shell theory
                          – PV22
                          Aug 11 at 17:47




                          +1 shell theory
                          – PV22
                          Aug 11 at 17:47










                          up vote
                          17
                          down vote













                          The Gravitational Constant is not uniform throughout space and time



                          Scientists have long speculated that some fundamental constants of the universe, like the speed of light or the fine structure constants, are varying through time.



                          Why not make it varying through space as well? This way a part of the galaxy can decay while the other parts are still bound by normal gravity.






                          share|improve this answer
















                          • 2




                            Notably, if the parameters vary through time, you've lost conservation of energy. If they vary across space, you've lost conservation of momentum. (Noether's Theorem means every conservation law is equivalent to a symmetry; if you lose one you lose the other.)
                            – Draconis
                            Aug 11 at 21:18














                          up vote
                          17
                          down vote













                          The Gravitational Constant is not uniform throughout space and time



                          Scientists have long speculated that some fundamental constants of the universe, like the speed of light or the fine structure constants, are varying through time.



                          Why not make it varying through space as well? This way a part of the galaxy can decay while the other parts are still bound by normal gravity.






                          share|improve this answer
















                          • 2




                            Notably, if the parameters vary through time, you've lost conservation of energy. If they vary across space, you've lost conservation of momentum. (Noether's Theorem means every conservation law is equivalent to a symmetry; if you lose one you lose the other.)
                            – Draconis
                            Aug 11 at 21:18












                          up vote
                          17
                          down vote










                          up vote
                          17
                          down vote









                          The Gravitational Constant is not uniform throughout space and time



                          Scientists have long speculated that some fundamental constants of the universe, like the speed of light or the fine structure constants, are varying through time.



                          Why not make it varying through space as well? This way a part of the galaxy can decay while the other parts are still bound by normal gravity.






                          share|improve this answer












                          The Gravitational Constant is not uniform throughout space and time



                          Scientists have long speculated that some fundamental constants of the universe, like the speed of light or the fine structure constants, are varying through time.



                          Why not make it varying through space as well? This way a part of the galaxy can decay while the other parts are still bound by normal gravity.







                          share|improve this answer












                          share|improve this answer



                          share|improve this answer










                          answered Aug 9 at 22:37









                          Henricus V.

                          31115




                          31115







                          • 2




                            Notably, if the parameters vary through time, you've lost conservation of energy. If they vary across space, you've lost conservation of momentum. (Noether's Theorem means every conservation law is equivalent to a symmetry; if you lose one you lose the other.)
                            – Draconis
                            Aug 11 at 21:18












                          • 2




                            Notably, if the parameters vary through time, you've lost conservation of energy. If they vary across space, you've lost conservation of momentum. (Noether's Theorem means every conservation law is equivalent to a symmetry; if you lose one you lose the other.)
                            – Draconis
                            Aug 11 at 21:18







                          2




                          2




                          Notably, if the parameters vary through time, you've lost conservation of energy. If they vary across space, you've lost conservation of momentum. (Noether's Theorem means every conservation law is equivalent to a symmetry; if you lose one you lose the other.)
                          – Draconis
                          Aug 11 at 21:18




                          Notably, if the parameters vary through time, you've lost conservation of energy. If they vary across space, you've lost conservation of momentum. (Noether's Theorem means every conservation law is equivalent to a symmetry; if you lose one you lose the other.)
                          – Draconis
                          Aug 11 at 21:18










                          up vote
                          9
                          down vote













                          Unexpectedly for all the community of scientists in the world, gravitons (the particles mediating gravitational force) have a finite life time, and they are starting to decay.



                          The more they decay, the more gravity is weakened. After a certain time gravity will no longer bind together masses, so unless they are electrically charged and can rely on good old Coulomb force, they are bound to separate.



                          Space will be filled with atoms traveling around, no longer aggregated.






                          share|improve this answer




















                          • This is a VERY interesting way of looking at it!
                            – JBH
                            Aug 9 at 19:53










                          • Also deeply depressing for the story, because the result would be inescapable. Kind of an On the Beach ending. "And in the universe, there was silence."
                            – jdunlop
                            Aug 9 at 20:07






                          • 7




                            "gravitons have a finite life time" - usually the particle life time is measured from its creation, and decaying force carriers would only manifest as a force that decays more than quadratically with distance. You'd need the life time to change over time. Do note that real-life gravity seems to have no such super-quadratic dropoff - and increasing it would crank up the expansion of the universe rather significantly. And in any case, we'd drift away from our Sun as a planet far before we'd drift away from our planet.as individual organisms, even without the increased expansion...
                            – John Dvorak
                            Aug 9 at 22:43










                          • @JohnDvorak Perhaps the gravity particles act more like air does for sound — carrying it, but not being created when gravitational force is emitted. However, all the particles (that I know of) we’ve encountered have decayed exponentially, making this seem somewhat unrealistic to me.
                            – J F
                            Aug 10 at 11:00














                          up vote
                          9
                          down vote













                          Unexpectedly for all the community of scientists in the world, gravitons (the particles mediating gravitational force) have a finite life time, and they are starting to decay.



                          The more they decay, the more gravity is weakened. After a certain time gravity will no longer bind together masses, so unless they are electrically charged and can rely on good old Coulomb force, they are bound to separate.



                          Space will be filled with atoms traveling around, no longer aggregated.






                          share|improve this answer




















                          • This is a VERY interesting way of looking at it!
                            – JBH
                            Aug 9 at 19:53










                          • Also deeply depressing for the story, because the result would be inescapable. Kind of an On the Beach ending. "And in the universe, there was silence."
                            – jdunlop
                            Aug 9 at 20:07






                          • 7




                            "gravitons have a finite life time" - usually the particle life time is measured from its creation, and decaying force carriers would only manifest as a force that decays more than quadratically with distance. You'd need the life time to change over time. Do note that real-life gravity seems to have no such super-quadratic dropoff - and increasing it would crank up the expansion of the universe rather significantly. And in any case, we'd drift away from our Sun as a planet far before we'd drift away from our planet.as individual organisms, even without the increased expansion...
                            – John Dvorak
                            Aug 9 at 22:43










                          • @JohnDvorak Perhaps the gravity particles act more like air does for sound — carrying it, but not being created when gravitational force is emitted. However, all the particles (that I know of) we’ve encountered have decayed exponentially, making this seem somewhat unrealistic to me.
                            – J F
                            Aug 10 at 11:00












                          up vote
                          9
                          down vote










                          up vote
                          9
                          down vote









                          Unexpectedly for all the community of scientists in the world, gravitons (the particles mediating gravitational force) have a finite life time, and they are starting to decay.



                          The more they decay, the more gravity is weakened. After a certain time gravity will no longer bind together masses, so unless they are electrically charged and can rely on good old Coulomb force, they are bound to separate.



                          Space will be filled with atoms traveling around, no longer aggregated.






                          share|improve this answer












                          Unexpectedly for all the community of scientists in the world, gravitons (the particles mediating gravitational force) have a finite life time, and they are starting to decay.



                          The more they decay, the more gravity is weakened. After a certain time gravity will no longer bind together masses, so unless they are electrically charged and can rely on good old Coulomb force, they are bound to separate.



                          Space will be filled with atoms traveling around, no longer aggregated.







                          share|improve this answer












                          share|improve this answer



                          share|improve this answer










                          answered Aug 9 at 19:26









                          L.Dutch♦

                          60.7k17142284




                          60.7k17142284











                          • This is a VERY interesting way of looking at it!
                            – JBH
                            Aug 9 at 19:53










                          • Also deeply depressing for the story, because the result would be inescapable. Kind of an On the Beach ending. "And in the universe, there was silence."
                            – jdunlop
                            Aug 9 at 20:07






                          • 7




                            "gravitons have a finite life time" - usually the particle life time is measured from its creation, and decaying force carriers would only manifest as a force that decays more than quadratically with distance. You'd need the life time to change over time. Do note that real-life gravity seems to have no such super-quadratic dropoff - and increasing it would crank up the expansion of the universe rather significantly. And in any case, we'd drift away from our Sun as a planet far before we'd drift away from our planet.as individual organisms, even without the increased expansion...
                            – John Dvorak
                            Aug 9 at 22:43










                          • @JohnDvorak Perhaps the gravity particles act more like air does for sound — carrying it, but not being created when gravitational force is emitted. However, all the particles (that I know of) we’ve encountered have decayed exponentially, making this seem somewhat unrealistic to me.
                            – J F
                            Aug 10 at 11:00
















                          • This is a VERY interesting way of looking at it!
                            – JBH
                            Aug 9 at 19:53










                          • Also deeply depressing for the story, because the result would be inescapable. Kind of an On the Beach ending. "And in the universe, there was silence."
                            – jdunlop
                            Aug 9 at 20:07






                          • 7




                            "gravitons have a finite life time" - usually the particle life time is measured from its creation, and decaying force carriers would only manifest as a force that decays more than quadratically with distance. You'd need the life time to change over time. Do note that real-life gravity seems to have no such super-quadratic dropoff - and increasing it would crank up the expansion of the universe rather significantly. And in any case, we'd drift away from our Sun as a planet far before we'd drift away from our planet.as individual organisms, even without the increased expansion...
                            – John Dvorak
                            Aug 9 at 22:43










                          • @JohnDvorak Perhaps the gravity particles act more like air does for sound — carrying it, but not being created when gravitational force is emitted. However, all the particles (that I know of) we’ve encountered have decayed exponentially, making this seem somewhat unrealistic to me.
                            – J F
                            Aug 10 at 11:00















                          This is a VERY interesting way of looking at it!
                          – JBH
                          Aug 9 at 19:53




                          This is a VERY interesting way of looking at it!
                          – JBH
                          Aug 9 at 19:53












                          Also deeply depressing for the story, because the result would be inescapable. Kind of an On the Beach ending. "And in the universe, there was silence."
                          – jdunlop
                          Aug 9 at 20:07




                          Also deeply depressing for the story, because the result would be inescapable. Kind of an On the Beach ending. "And in the universe, there was silence."
                          – jdunlop
                          Aug 9 at 20:07




                          7




                          7




                          "gravitons have a finite life time" - usually the particle life time is measured from its creation, and decaying force carriers would only manifest as a force that decays more than quadratically with distance. You'd need the life time to change over time. Do note that real-life gravity seems to have no such super-quadratic dropoff - and increasing it would crank up the expansion of the universe rather significantly. And in any case, we'd drift away from our Sun as a planet far before we'd drift away from our planet.as individual organisms, even without the increased expansion...
                          – John Dvorak
                          Aug 9 at 22:43




                          "gravitons have a finite life time" - usually the particle life time is measured from its creation, and decaying force carriers would only manifest as a force that decays more than quadratically with distance. You'd need the life time to change over time. Do note that real-life gravity seems to have no such super-quadratic dropoff - and increasing it would crank up the expansion of the universe rather significantly. And in any case, we'd drift away from our Sun as a planet far before we'd drift away from our planet.as individual organisms, even without the increased expansion...
                          – John Dvorak
                          Aug 9 at 22:43












                          @JohnDvorak Perhaps the gravity particles act more like air does for sound — carrying it, but not being created when gravitational force is emitted. However, all the particles (that I know of) we’ve encountered have decayed exponentially, making this seem somewhat unrealistic to me.
                          – J F
                          Aug 10 at 11:00




                          @JohnDvorak Perhaps the gravity particles act more like air does for sound — carrying it, but not being created when gravitational force is emitted. However, all the particles (that I know of) we’ve encountered have decayed exponentially, making this seem somewhat unrealistic to me.
                          – J F
                          Aug 10 at 11:00










                          up vote
                          6
                          down vote













                          In order of decreasing viability (all of which are still super low):



                          • The simulation hypothesis, that we are all living in a simulation, is really your best bet here. If gravity were capable of failing one would expect to see stars sort of just blinking out of existence every now and then, which we don't. That being said if we are living in a simulation (which is quite likely) then really the simulators can do whatever they want with physics.


                          • Aliens, "pull the fabric of space-time taught." does it mean anything? not really no. is it intuitive? yes.


                          • Act of god. plain simple, always in style. I'd go with this explaination myself


                          • Freak variation in the gravitational constant. https://en.wikipedia.org/wiki/Time-variation_of_fundamental_constants. it's not certain that G the graviational constant of the universe is always well constant, Some theories hold that it varies. If it suddenly varies down (possibly even as a result of human tampering, probably not though.) gravity turns "off" simple as that. that being said this wipes out the universe not just earth. Bare in mind suddenly in this context could still be upwards of a million years or just a few seconds so it gives you a lot of narrative freedom.


                          • Presence of large numbers of "Anti-Gravitons". the Graviton (a hypothetical particle that carries the force of gravity) sometimes has an even more hypothetical counter part called the anti graviton aleins fire a bunch of theese at us and they react in unexpected ways with currently existing gravitons destroying them and sending the excess energy away as gravitational waves.


                          • Vacuum decay? there is a thing called Vacuum decay, most likely it will simply wipe out everything if real however there is a INCREDIBLY small chance that it simply re-writes the laws of reality instead.


                          All of these answers (except Act of god & simulation, which have their own problems) are pseudo-scientific to the extreme and highly unlikely to actually work. If you go for simulation, god, or either of the aliens options you can always also put gravity back before the earth if fully destroyed leaving people left on earth with just a bunch of earth-quakes and a new outlook on life to deal with, if that's you narrative intention that is.






                          share|improve this answer






















                          • I haven't fully decided on an ending, so anything is possible... obviously life as everyone would know it would end... I have toyed with aliens or dimensional sources, but mostly as a way to narrate. As if all the story lines I'm presenting are part of a massive archeological find, and some being, whether an alien civilization or humans that somehow escape come back to piece together events.
                            – SEK1977
                            Aug 9 at 20:25






                          • 2




                            Do you mean pull it "taut"?
                            – Azor Ahai
                            Aug 9 at 23:17










                          • Well of course i'm speaking nonsense when i say "pull it taut" however there is a demonstration of the behaviour of gravity where you place two balls on rubber sheet and they move together in this demonstration pulling the sheet taut will cause gravity to cease existing. of course in reality it's meaningless and more or less impossible but it makes more plausible explanation than a lot of stuff in sci-fi. in closest real world terms it would be the equivalent of using artificial gravity to exactly cancel out earths gravity
                            – Ummdustry
                            Aug 10 at 7:26














                          up vote
                          6
                          down vote













                          In order of decreasing viability (all of which are still super low):



                          • The simulation hypothesis, that we are all living in a simulation, is really your best bet here. If gravity were capable of failing one would expect to see stars sort of just blinking out of existence every now and then, which we don't. That being said if we are living in a simulation (which is quite likely) then really the simulators can do whatever they want with physics.


                          • Aliens, "pull the fabric of space-time taught." does it mean anything? not really no. is it intuitive? yes.


                          • Act of god. plain simple, always in style. I'd go with this explaination myself


                          • Freak variation in the gravitational constant. https://en.wikipedia.org/wiki/Time-variation_of_fundamental_constants. it's not certain that G the graviational constant of the universe is always well constant, Some theories hold that it varies. If it suddenly varies down (possibly even as a result of human tampering, probably not though.) gravity turns "off" simple as that. that being said this wipes out the universe not just earth. Bare in mind suddenly in this context could still be upwards of a million years or just a few seconds so it gives you a lot of narrative freedom.


                          • Presence of large numbers of "Anti-Gravitons". the Graviton (a hypothetical particle that carries the force of gravity) sometimes has an even more hypothetical counter part called the anti graviton aleins fire a bunch of theese at us and they react in unexpected ways with currently existing gravitons destroying them and sending the excess energy away as gravitational waves.


                          • Vacuum decay? there is a thing called Vacuum decay, most likely it will simply wipe out everything if real however there is a INCREDIBLY small chance that it simply re-writes the laws of reality instead.


                          All of these answers (except Act of god & simulation, which have their own problems) are pseudo-scientific to the extreme and highly unlikely to actually work. If you go for simulation, god, or either of the aliens options you can always also put gravity back before the earth if fully destroyed leaving people left on earth with just a bunch of earth-quakes and a new outlook on life to deal with, if that's you narrative intention that is.






                          share|improve this answer






















                          • I haven't fully decided on an ending, so anything is possible... obviously life as everyone would know it would end... I have toyed with aliens or dimensional sources, but mostly as a way to narrate. As if all the story lines I'm presenting are part of a massive archeological find, and some being, whether an alien civilization or humans that somehow escape come back to piece together events.
                            – SEK1977
                            Aug 9 at 20:25






                          • 2




                            Do you mean pull it "taut"?
                            – Azor Ahai
                            Aug 9 at 23:17










                          • Well of course i'm speaking nonsense when i say "pull it taut" however there is a demonstration of the behaviour of gravity where you place two balls on rubber sheet and they move together in this demonstration pulling the sheet taut will cause gravity to cease existing. of course in reality it's meaningless and more or less impossible but it makes more plausible explanation than a lot of stuff in sci-fi. in closest real world terms it would be the equivalent of using artificial gravity to exactly cancel out earths gravity
                            – Ummdustry
                            Aug 10 at 7:26












                          up vote
                          6
                          down vote










                          up vote
                          6
                          down vote









                          In order of decreasing viability (all of which are still super low):



                          • The simulation hypothesis, that we are all living in a simulation, is really your best bet here. If gravity were capable of failing one would expect to see stars sort of just blinking out of existence every now and then, which we don't. That being said if we are living in a simulation (which is quite likely) then really the simulators can do whatever they want with physics.


                          • Aliens, "pull the fabric of space-time taught." does it mean anything? not really no. is it intuitive? yes.


                          • Act of god. plain simple, always in style. I'd go with this explaination myself


                          • Freak variation in the gravitational constant. https://en.wikipedia.org/wiki/Time-variation_of_fundamental_constants. it's not certain that G the graviational constant of the universe is always well constant, Some theories hold that it varies. If it suddenly varies down (possibly even as a result of human tampering, probably not though.) gravity turns "off" simple as that. that being said this wipes out the universe not just earth. Bare in mind suddenly in this context could still be upwards of a million years or just a few seconds so it gives you a lot of narrative freedom.


                          • Presence of large numbers of "Anti-Gravitons". the Graviton (a hypothetical particle that carries the force of gravity) sometimes has an even more hypothetical counter part called the anti graviton aleins fire a bunch of theese at us and they react in unexpected ways with currently existing gravitons destroying them and sending the excess energy away as gravitational waves.


                          • Vacuum decay? there is a thing called Vacuum decay, most likely it will simply wipe out everything if real however there is a INCREDIBLY small chance that it simply re-writes the laws of reality instead.


                          All of these answers (except Act of god & simulation, which have their own problems) are pseudo-scientific to the extreme and highly unlikely to actually work. If you go for simulation, god, or either of the aliens options you can always also put gravity back before the earth if fully destroyed leaving people left on earth with just a bunch of earth-quakes and a new outlook on life to deal with, if that's you narrative intention that is.






                          share|improve this answer














                          In order of decreasing viability (all of which are still super low):



                          • The simulation hypothesis, that we are all living in a simulation, is really your best bet here. If gravity were capable of failing one would expect to see stars sort of just blinking out of existence every now and then, which we don't. That being said if we are living in a simulation (which is quite likely) then really the simulators can do whatever they want with physics.


                          • Aliens, "pull the fabric of space-time taught." does it mean anything? not really no. is it intuitive? yes.


                          • Act of god. plain simple, always in style. I'd go with this explaination myself


                          • Freak variation in the gravitational constant. https://en.wikipedia.org/wiki/Time-variation_of_fundamental_constants. it's not certain that G the graviational constant of the universe is always well constant, Some theories hold that it varies. If it suddenly varies down (possibly even as a result of human tampering, probably not though.) gravity turns "off" simple as that. that being said this wipes out the universe not just earth. Bare in mind suddenly in this context could still be upwards of a million years or just a few seconds so it gives you a lot of narrative freedom.


                          • Presence of large numbers of "Anti-Gravitons". the Graviton (a hypothetical particle that carries the force of gravity) sometimes has an even more hypothetical counter part called the anti graviton aleins fire a bunch of theese at us and they react in unexpected ways with currently existing gravitons destroying them and sending the excess energy away as gravitational waves.


                          • Vacuum decay? there is a thing called Vacuum decay, most likely it will simply wipe out everything if real however there is a INCREDIBLY small chance that it simply re-writes the laws of reality instead.


                          All of these answers (except Act of god & simulation, which have their own problems) are pseudo-scientific to the extreme and highly unlikely to actually work. If you go for simulation, god, or either of the aliens options you can always also put gravity back before the earth if fully destroyed leaving people left on earth with just a bunch of earth-quakes and a new outlook on life to deal with, if that's you narrative intention that is.







                          share|improve this answer














                          share|improve this answer



                          share|improve this answer








                          edited Aug 9 at 19:55









                          JBH

                          32.2k578153




                          32.2k578153










                          answered Aug 9 at 19:30









                          Ummdustry

                          2,428513




                          2,428513











                          • I haven't fully decided on an ending, so anything is possible... obviously life as everyone would know it would end... I have toyed with aliens or dimensional sources, but mostly as a way to narrate. As if all the story lines I'm presenting are part of a massive archeological find, and some being, whether an alien civilization or humans that somehow escape come back to piece together events.
                            – SEK1977
                            Aug 9 at 20:25






                          • 2




                            Do you mean pull it "taut"?
                            – Azor Ahai
                            Aug 9 at 23:17










                          • Well of course i'm speaking nonsense when i say "pull it taut" however there is a demonstration of the behaviour of gravity where you place two balls on rubber sheet and they move together in this demonstration pulling the sheet taut will cause gravity to cease existing. of course in reality it's meaningless and more or less impossible but it makes more plausible explanation than a lot of stuff in sci-fi. in closest real world terms it would be the equivalent of using artificial gravity to exactly cancel out earths gravity
                            – Ummdustry
                            Aug 10 at 7:26
















                          • I haven't fully decided on an ending, so anything is possible... obviously life as everyone would know it would end... I have toyed with aliens or dimensional sources, but mostly as a way to narrate. As if all the story lines I'm presenting are part of a massive archeological find, and some being, whether an alien civilization or humans that somehow escape come back to piece together events.
                            – SEK1977
                            Aug 9 at 20:25






                          • 2




                            Do you mean pull it "taut"?
                            – Azor Ahai
                            Aug 9 at 23:17










                          • Well of course i'm speaking nonsense when i say "pull it taut" however there is a demonstration of the behaviour of gravity where you place two balls on rubber sheet and they move together in this demonstration pulling the sheet taut will cause gravity to cease existing. of course in reality it's meaningless and more or less impossible but it makes more plausible explanation than a lot of stuff in sci-fi. in closest real world terms it would be the equivalent of using artificial gravity to exactly cancel out earths gravity
                            – Ummdustry
                            Aug 10 at 7:26















                          I haven't fully decided on an ending, so anything is possible... obviously life as everyone would know it would end... I have toyed with aliens or dimensional sources, but mostly as a way to narrate. As if all the story lines I'm presenting are part of a massive archeological find, and some being, whether an alien civilization or humans that somehow escape come back to piece together events.
                          – SEK1977
                          Aug 9 at 20:25




                          I haven't fully decided on an ending, so anything is possible... obviously life as everyone would know it would end... I have toyed with aliens or dimensional sources, but mostly as a way to narrate. As if all the story lines I'm presenting are part of a massive archeological find, and some being, whether an alien civilization or humans that somehow escape come back to piece together events.
                          – SEK1977
                          Aug 9 at 20:25




                          2




                          2




                          Do you mean pull it "taut"?
                          – Azor Ahai
                          Aug 9 at 23:17




                          Do you mean pull it "taut"?
                          – Azor Ahai
                          Aug 9 at 23:17












                          Well of course i'm speaking nonsense when i say "pull it taut" however there is a demonstration of the behaviour of gravity where you place two balls on rubber sheet and they move together in this demonstration pulling the sheet taut will cause gravity to cease existing. of course in reality it's meaningless and more or less impossible but it makes more plausible explanation than a lot of stuff in sci-fi. in closest real world terms it would be the equivalent of using artificial gravity to exactly cancel out earths gravity
                          – Ummdustry
                          Aug 10 at 7:26




                          Well of course i'm speaking nonsense when i say "pull it taut" however there is a demonstration of the behaviour of gravity where you place two balls on rubber sheet and they move together in this demonstration pulling the sheet taut will cause gravity to cease existing. of course in reality it's meaningless and more or less impossible but it makes more plausible explanation than a lot of stuff in sci-fi. in closest real world terms it would be the equivalent of using artificial gravity to exactly cancel out earths gravity
                          – Ummdustry
                          Aug 10 at 7:26










                          up vote
                          5
                          down vote













                          It sounds like the big rip theory answers your question. Space-time expands and therefore the entire universe expands. The theory says the expansion will eventually overcome gravity.
                          https://en.m.wikipedia.org/wiki/Big_Rip






                          share|improve this answer




















                          • I hadn't heard of this one, thank you.
                            – SEK1977
                            Aug 9 at 20:28














                          up vote
                          5
                          down vote













                          It sounds like the big rip theory answers your question. Space-time expands and therefore the entire universe expands. The theory says the expansion will eventually overcome gravity.
                          https://en.m.wikipedia.org/wiki/Big_Rip






                          share|improve this answer




















                          • I hadn't heard of this one, thank you.
                            – SEK1977
                            Aug 9 at 20:28












                          up vote
                          5
                          down vote










                          up vote
                          5
                          down vote









                          It sounds like the big rip theory answers your question. Space-time expands and therefore the entire universe expands. The theory says the expansion will eventually overcome gravity.
                          https://en.m.wikipedia.org/wiki/Big_Rip






                          share|improve this answer












                          It sounds like the big rip theory answers your question. Space-time expands and therefore the entire universe expands. The theory says the expansion will eventually overcome gravity.
                          https://en.m.wikipedia.org/wiki/Big_Rip







                          share|improve this answer












                          share|improve this answer



                          share|improve this answer










                          answered Aug 9 at 19:42









                          Christmas Snow

                          1,38829




                          1,38829











                          • I hadn't heard of this one, thank you.
                            – SEK1977
                            Aug 9 at 20:28
















                          • I hadn't heard of this one, thank you.
                            – SEK1977
                            Aug 9 at 20:28















                          I hadn't heard of this one, thank you.
                          – SEK1977
                          Aug 9 at 20:28




                          I hadn't heard of this one, thank you.
                          – SEK1977
                          Aug 9 at 20:28










                          up vote
                          4
                          down vote













                          To diminish gravity, you have to diminish mass. Currently the only way we know how to do that is to convert it explosively into energy. I don't think that will work for you here because if you converted enough of Earth that we would notice the lower gravity, the created energy would probably melt the crust and everyone on it. That makes for a short and tragic story.



                          What you probably need here is a dimensional doorway, but that is way beyond our hard science, so the rest of this answer will be in violation of your "science-based" tag.



                          If a dimensional doorway existed at the core of our planet, and if a steady stream of core material poured through that doorway and into another plane of existence, and if no reciprocal mass or energy came back through the doorway into our reality, then the planet would slowly (or not so slowly) loose mass. As this happened, the gravity would diminish, but so would the rotational velocity (making our days longer) and the magnetic field (making our days a lot more interesting, i.e. nuclear sunburn). It would get you what you are looking for in terms of failing gravity but the side effects might pose more urgent threats than the sudden weight loss.






                          share|improve this answer






















                          • Interestingly, covering mass to energy would not reduce its gravitational pull, as mass and energy are equivalent. They are actually represented by the same term in the Einstein field equations.
                            – Joel Keene
                            Aug 10 at 12:08










                          • @JoelKeene, Really? That is non-intuitive. So in a super nova, as a wave of energy propagates out in all directions, that energy has mass and therefore exudes gravity (in addition to its concussive force). I didn't think that energy had mass. The more I hang out on this site, the more I realize that Einstein was the moment when scientists and laymen became separate species.
                            – Henry Taylor
                            Aug 10 at 13:11














                          up vote
                          4
                          down vote













                          To diminish gravity, you have to diminish mass. Currently the only way we know how to do that is to convert it explosively into energy. I don't think that will work for you here because if you converted enough of Earth that we would notice the lower gravity, the created energy would probably melt the crust and everyone on it. That makes for a short and tragic story.



                          What you probably need here is a dimensional doorway, but that is way beyond our hard science, so the rest of this answer will be in violation of your "science-based" tag.



                          If a dimensional doorway existed at the core of our planet, and if a steady stream of core material poured through that doorway and into another plane of existence, and if no reciprocal mass or energy came back through the doorway into our reality, then the planet would slowly (or not so slowly) loose mass. As this happened, the gravity would diminish, but so would the rotational velocity (making our days longer) and the magnetic field (making our days a lot more interesting, i.e. nuclear sunburn). It would get you what you are looking for in terms of failing gravity but the side effects might pose more urgent threats than the sudden weight loss.






                          share|improve this answer






















                          • Interestingly, covering mass to energy would not reduce its gravitational pull, as mass and energy are equivalent. They are actually represented by the same term in the Einstein field equations.
                            – Joel Keene
                            Aug 10 at 12:08










                          • @JoelKeene, Really? That is non-intuitive. So in a super nova, as a wave of energy propagates out in all directions, that energy has mass and therefore exudes gravity (in addition to its concussive force). I didn't think that energy had mass. The more I hang out on this site, the more I realize that Einstein was the moment when scientists and laymen became separate species.
                            – Henry Taylor
                            Aug 10 at 13:11












                          up vote
                          4
                          down vote










                          up vote
                          4
                          down vote









                          To diminish gravity, you have to diminish mass. Currently the only way we know how to do that is to convert it explosively into energy. I don't think that will work for you here because if you converted enough of Earth that we would notice the lower gravity, the created energy would probably melt the crust and everyone on it. That makes for a short and tragic story.



                          What you probably need here is a dimensional doorway, but that is way beyond our hard science, so the rest of this answer will be in violation of your "science-based" tag.



                          If a dimensional doorway existed at the core of our planet, and if a steady stream of core material poured through that doorway and into another plane of existence, and if no reciprocal mass or energy came back through the doorway into our reality, then the planet would slowly (or not so slowly) loose mass. As this happened, the gravity would diminish, but so would the rotational velocity (making our days longer) and the magnetic field (making our days a lot more interesting, i.e. nuclear sunburn). It would get you what you are looking for in terms of failing gravity but the side effects might pose more urgent threats than the sudden weight loss.






                          share|improve this answer














                          To diminish gravity, you have to diminish mass. Currently the only way we know how to do that is to convert it explosively into energy. I don't think that will work for you here because if you converted enough of Earth that we would notice the lower gravity, the created energy would probably melt the crust and everyone on it. That makes for a short and tragic story.



                          What you probably need here is a dimensional doorway, but that is way beyond our hard science, so the rest of this answer will be in violation of your "science-based" tag.



                          If a dimensional doorway existed at the core of our planet, and if a steady stream of core material poured through that doorway and into another plane of existence, and if no reciprocal mass or energy came back through the doorway into our reality, then the planet would slowly (or not so slowly) loose mass. As this happened, the gravity would diminish, but so would the rotational velocity (making our days longer) and the magnetic field (making our days a lot more interesting, i.e. nuclear sunburn). It would get you what you are looking for in terms of failing gravity but the side effects might pose more urgent threats than the sudden weight loss.







                          share|improve this answer














                          share|improve this answer



                          share|improve this answer








                          edited Aug 9 at 20:37

























                          answered Aug 9 at 19:16









                          Henry Taylor

                          41.6k764150




                          41.6k764150











                          • Interestingly, covering mass to energy would not reduce its gravitational pull, as mass and energy are equivalent. They are actually represented by the same term in the Einstein field equations.
                            – Joel Keene
                            Aug 10 at 12:08










                          • @JoelKeene, Really? That is non-intuitive. So in a super nova, as a wave of energy propagates out in all directions, that energy has mass and therefore exudes gravity (in addition to its concussive force). I didn't think that energy had mass. The more I hang out on this site, the more I realize that Einstein was the moment when scientists and laymen became separate species.
                            – Henry Taylor
                            Aug 10 at 13:11
















                          • Interestingly, covering mass to energy would not reduce its gravitational pull, as mass and energy are equivalent. They are actually represented by the same term in the Einstein field equations.
                            – Joel Keene
                            Aug 10 at 12:08










                          • @JoelKeene, Really? That is non-intuitive. So in a super nova, as a wave of energy propagates out in all directions, that energy has mass and therefore exudes gravity (in addition to its concussive force). I didn't think that energy had mass. The more I hang out on this site, the more I realize that Einstein was the moment when scientists and laymen became separate species.
                            – Henry Taylor
                            Aug 10 at 13:11















                          Interestingly, covering mass to energy would not reduce its gravitational pull, as mass and energy are equivalent. They are actually represented by the same term in the Einstein field equations.
                          – Joel Keene
                          Aug 10 at 12:08




                          Interestingly, covering mass to energy would not reduce its gravitational pull, as mass and energy are equivalent. They are actually represented by the same term in the Einstein field equations.
                          – Joel Keene
                          Aug 10 at 12:08












                          @JoelKeene, Really? That is non-intuitive. So in a super nova, as a wave of energy propagates out in all directions, that energy has mass and therefore exudes gravity (in addition to its concussive force). I didn't think that energy had mass. The more I hang out on this site, the more I realize that Einstein was the moment when scientists and laymen became separate species.
                          – Henry Taylor
                          Aug 10 at 13:11




                          @JoelKeene, Really? That is non-intuitive. So in a super nova, as a wave of energy propagates out in all directions, that energy has mass and therefore exudes gravity (in addition to its concussive force). I didn't think that energy had mass. The more I hang out on this site, the more I realize that Einstein was the moment when scientists and laymen became separate species.
                          – Henry Taylor
                          Aug 10 at 13:11










                          up vote
                          3
                          down vote













                          Some ideas:



                          • Space is warped, and Earth's gravity well intersects with something on the other side of the warp. The space where our planet once sat is becoming a wormhole, and gravity is failing from our perspective as a result. See the illustration below. This idea is my favorite on this list

                          • After the largest underground nuclear bomb test in history, or maybe after another exciting and seemingly innocuous experiment at CERN, the small forces keeping atoms together in the earth's crust have ceased to function normally. The earth is losing mass to radiation at an alarming rate. In some parts of the world, people are dying from radiation sickness; in other parts, people are unable to reproduce, and in other parts the radiation exposure is minimal but the gravitational effects are nonetheless observable. Furthermore, this loss of mass results in large cave-ins, and cities falling into the ground like in Centralia PA.

                          • Mass hysteria. It's not really happening, just everybody thinks it is.

                          • Earth's gravity has been slowly dying for a long time, but the change was imperceptibly small until now. Now the change is accelerating asymptotically. It only seems to be affecting our planet, but really this is just a stage in the lifecycle of every planet, and we haven't observed it before.

                          • Beings from another spacial dimension create some 5th dimensional equivalent to a dyson-sphere around our planet.

                          • Beings from this spacial dimension create an actual dyson-sphere around our planet

                          Hope this helps!






                          share|improve this answer
















                          • 2




                            Mass hysteria is intriguing. It could open some more psychology to the story.
                            – SEK1977
                            Aug 9 at 20:26














                          up vote
                          3
                          down vote













                          Some ideas:



                          • Space is warped, and Earth's gravity well intersects with something on the other side of the warp. The space where our planet once sat is becoming a wormhole, and gravity is failing from our perspective as a result. See the illustration below. This idea is my favorite on this list

                          • After the largest underground nuclear bomb test in history, or maybe after another exciting and seemingly innocuous experiment at CERN, the small forces keeping atoms together in the earth's crust have ceased to function normally. The earth is losing mass to radiation at an alarming rate. In some parts of the world, people are dying from radiation sickness; in other parts, people are unable to reproduce, and in other parts the radiation exposure is minimal but the gravitational effects are nonetheless observable. Furthermore, this loss of mass results in large cave-ins, and cities falling into the ground like in Centralia PA.

                          • Mass hysteria. It's not really happening, just everybody thinks it is.

                          • Earth's gravity has been slowly dying for a long time, but the change was imperceptibly small until now. Now the change is accelerating asymptotically. It only seems to be affecting our planet, but really this is just a stage in the lifecycle of every planet, and we haven't observed it before.

                          • Beings from another spacial dimension create some 5th dimensional equivalent to a dyson-sphere around our planet.

                          • Beings from this spacial dimension create an actual dyson-sphere around our planet

                          Hope this helps!






                          share|improve this answer
















                          • 2




                            Mass hysteria is intriguing. It could open some more psychology to the story.
                            – SEK1977
                            Aug 9 at 20:26












                          up vote
                          3
                          down vote










                          up vote
                          3
                          down vote









                          Some ideas:



                          • Space is warped, and Earth's gravity well intersects with something on the other side of the warp. The space where our planet once sat is becoming a wormhole, and gravity is failing from our perspective as a result. See the illustration below. This idea is my favorite on this list

                          • After the largest underground nuclear bomb test in history, or maybe after another exciting and seemingly innocuous experiment at CERN, the small forces keeping atoms together in the earth's crust have ceased to function normally. The earth is losing mass to radiation at an alarming rate. In some parts of the world, people are dying from radiation sickness; in other parts, people are unable to reproduce, and in other parts the radiation exposure is minimal but the gravitational effects are nonetheless observable. Furthermore, this loss of mass results in large cave-ins, and cities falling into the ground like in Centralia PA.

                          • Mass hysteria. It's not really happening, just everybody thinks it is.

                          • Earth's gravity has been slowly dying for a long time, but the change was imperceptibly small until now. Now the change is accelerating asymptotically. It only seems to be affecting our planet, but really this is just a stage in the lifecycle of every planet, and we haven't observed it before.

                          • Beings from another spacial dimension create some 5th dimensional equivalent to a dyson-sphere around our planet.

                          • Beings from this spacial dimension create an actual dyson-sphere around our planet

                          Hope this helps!






                          share|improve this answer












                          Some ideas:



                          • Space is warped, and Earth's gravity well intersects with something on the other side of the warp. The space where our planet once sat is becoming a wormhole, and gravity is failing from our perspective as a result. See the illustration below. This idea is my favorite on this list

                          • After the largest underground nuclear bomb test in history, or maybe after another exciting and seemingly innocuous experiment at CERN, the small forces keeping atoms together in the earth's crust have ceased to function normally. The earth is losing mass to radiation at an alarming rate. In some parts of the world, people are dying from radiation sickness; in other parts, people are unable to reproduce, and in other parts the radiation exposure is minimal but the gravitational effects are nonetheless observable. Furthermore, this loss of mass results in large cave-ins, and cities falling into the ground like in Centralia PA.

                          • Mass hysteria. It's not really happening, just everybody thinks it is.

                          • Earth's gravity has been slowly dying for a long time, but the change was imperceptibly small until now. Now the change is accelerating asymptotically. It only seems to be affecting our planet, but really this is just a stage in the lifecycle of every planet, and we haven't observed it before.

                          • Beings from another spacial dimension create some 5th dimensional equivalent to a dyson-sphere around our planet.

                          • Beings from this spacial dimension create an actual dyson-sphere around our planet

                          Hope this helps!







                          share|improve this answer












                          share|improve this answer



                          share|improve this answer










                          answered Aug 9 at 19:57









                          boxcartenant

                          1,812116




                          1,812116







                          • 2




                            Mass hysteria is intriguing. It could open some more psychology to the story.
                            – SEK1977
                            Aug 9 at 20:26












                          • 2




                            Mass hysteria is intriguing. It could open some more psychology to the story.
                            – SEK1977
                            Aug 9 at 20:26







                          2




                          2




                          Mass hysteria is intriguing. It could open some more psychology to the story.
                          – SEK1977
                          Aug 9 at 20:26




                          Mass hysteria is intriguing. It could open some more psychology to the story.
                          – SEK1977
                          Aug 9 at 20:26










                          up vote
                          3
                          down vote













                          Your best bet is to use gravitons, the quantum particle/fundamental force-carrying particle of gravity. Dark matter is also a good candidate. Gravitons are similar to photons, the quantum particle of the electromagnetic force (also responsible for giving us light). Like Photons, gravitons are thought to have zero rest mass. This means gravitons travel forever at the speed of light and do not decay. However, gravitons could actually have a very small rest mass which would theoretically allow them to decay into other (lighter) particles.



                          Here are some options:



                          • Gravitons, which have some infinitesimal but nonzero rest mass, are finally starting to decay en-mass into other force-carrying particles such as photons or gluons.


                          • Society is emulated within a computer and is failing to floating point arithmetic errors. For instance in Javascript .1 + .2 = 0.30000000000000004. For instance plank's constant could be wrong.


                          • Dark Matter/Dark Energy - Physicists today observe large objects being attracted to certain areas of space that appear to be empty. Since we cannot observe what is attracting matter to these places other than that the attraction is happening via the gravitational force, we assume there is some massive body that is invisible to detection attracting everything. This scenario gives you the greatest flexibility since we really don't understand too much about Dark Matter/Energy. You could make it another form of life that exists in alternate dimensions or just have dark matter and energy be moving around the universe causing havoc.






                          share|improve this answer
























                            up vote
                            3
                            down vote













                            Your best bet is to use gravitons, the quantum particle/fundamental force-carrying particle of gravity. Dark matter is also a good candidate. Gravitons are similar to photons, the quantum particle of the electromagnetic force (also responsible for giving us light). Like Photons, gravitons are thought to have zero rest mass. This means gravitons travel forever at the speed of light and do not decay. However, gravitons could actually have a very small rest mass which would theoretically allow them to decay into other (lighter) particles.



                            Here are some options:



                            • Gravitons, which have some infinitesimal but nonzero rest mass, are finally starting to decay en-mass into other force-carrying particles such as photons or gluons.


                            • Society is emulated within a computer and is failing to floating point arithmetic errors. For instance in Javascript .1 + .2 = 0.30000000000000004. For instance plank's constant could be wrong.


                            • Dark Matter/Dark Energy - Physicists today observe large objects being attracted to certain areas of space that appear to be empty. Since we cannot observe what is attracting matter to these places other than that the attraction is happening via the gravitational force, we assume there is some massive body that is invisible to detection attracting everything. This scenario gives you the greatest flexibility since we really don't understand too much about Dark Matter/Energy. You could make it another form of life that exists in alternate dimensions or just have dark matter and energy be moving around the universe causing havoc.






                            share|improve this answer






















                              up vote
                              3
                              down vote










                              up vote
                              3
                              down vote









                              Your best bet is to use gravitons, the quantum particle/fundamental force-carrying particle of gravity. Dark matter is also a good candidate. Gravitons are similar to photons, the quantum particle of the electromagnetic force (also responsible for giving us light). Like Photons, gravitons are thought to have zero rest mass. This means gravitons travel forever at the speed of light and do not decay. However, gravitons could actually have a very small rest mass which would theoretically allow them to decay into other (lighter) particles.



                              Here are some options:



                              • Gravitons, which have some infinitesimal but nonzero rest mass, are finally starting to decay en-mass into other force-carrying particles such as photons or gluons.


                              • Society is emulated within a computer and is failing to floating point arithmetic errors. For instance in Javascript .1 + .2 = 0.30000000000000004. For instance plank's constant could be wrong.


                              • Dark Matter/Dark Energy - Physicists today observe large objects being attracted to certain areas of space that appear to be empty. Since we cannot observe what is attracting matter to these places other than that the attraction is happening via the gravitational force, we assume there is some massive body that is invisible to detection attracting everything. This scenario gives you the greatest flexibility since we really don't understand too much about Dark Matter/Energy. You could make it another form of life that exists in alternate dimensions or just have dark matter and energy be moving around the universe causing havoc.






                              share|improve this answer












                              Your best bet is to use gravitons, the quantum particle/fundamental force-carrying particle of gravity. Dark matter is also a good candidate. Gravitons are similar to photons, the quantum particle of the electromagnetic force (also responsible for giving us light). Like Photons, gravitons are thought to have zero rest mass. This means gravitons travel forever at the speed of light and do not decay. However, gravitons could actually have a very small rest mass which would theoretically allow them to decay into other (lighter) particles.



                              Here are some options:



                              • Gravitons, which have some infinitesimal but nonzero rest mass, are finally starting to decay en-mass into other force-carrying particles such as photons or gluons.


                              • Society is emulated within a computer and is failing to floating point arithmetic errors. For instance in Javascript .1 + .2 = 0.30000000000000004. For instance plank's constant could be wrong.


                              • Dark Matter/Dark Energy - Physicists today observe large objects being attracted to certain areas of space that appear to be empty. Since we cannot observe what is attracting matter to these places other than that the attraction is happening via the gravitational force, we assume there is some massive body that is invisible to detection attracting everything. This scenario gives you the greatest flexibility since we really don't understand too much about Dark Matter/Energy. You could make it another form of life that exists in alternate dimensions or just have dark matter and energy be moving around the universe causing havoc.







                              share|improve this answer












                              share|improve this answer



                              share|improve this answer










                              answered Aug 9 at 22:35









                              Isaac Flaum

                              311




                              311




















                                  up vote
                                  3
                                  down vote













                                  Gravity As A Wave



                                  Currently we don't know if gravity is a wave or a particle, but I've read it theorized that it's both, similar to photons. If we assume that gravity can exist as a wave, then perhaps a wave that happen to be the inverse of the gravitational wave signature of Earth's gravity well could cancel out that waveform an therefore nullify gravity. I'm not sure if destructive interference would actually apply to gravity waves, but maybe!



                                  I would expect something like that to be temporary or fleeting, but there are plenty of cosmological events that might create gravity waves. It could even be a failed human experiment or intentional attack from an outside force.



                                  Destructive Interference






                                  share|improve this answer
















                                  • 3




                                    Welcome to Worldbuilding! Just so you're aware, they have experimentally verified that gravitational waves (i.e. what it looks like you're talking about) do exist. It's a bit more complicated than this but the general idea of putting something that gives an equally strong gravitational field but with the force in the opposite direction would cancel out Earth's gravity at a point/small region. Gravitational waves are just a perturbation and those emitted by Earth are negligibly weak, but if you had a pair of merging black holes relatively nearby, then indeed, who knows what the result would be?
                                    – Mithrandir24601
                                    Aug 9 at 23:27














                                  up vote
                                  3
                                  down vote













                                  Gravity As A Wave



                                  Currently we don't know if gravity is a wave or a particle, but I've read it theorized that it's both, similar to photons. If we assume that gravity can exist as a wave, then perhaps a wave that happen to be the inverse of the gravitational wave signature of Earth's gravity well could cancel out that waveform an therefore nullify gravity. I'm not sure if destructive interference would actually apply to gravity waves, but maybe!



                                  I would expect something like that to be temporary or fleeting, but there are plenty of cosmological events that might create gravity waves. It could even be a failed human experiment or intentional attack from an outside force.



                                  Destructive Interference






                                  share|improve this answer
















                                  • 3




                                    Welcome to Worldbuilding! Just so you're aware, they have experimentally verified that gravitational waves (i.e. what it looks like you're talking about) do exist. It's a bit more complicated than this but the general idea of putting something that gives an equally strong gravitational field but with the force in the opposite direction would cancel out Earth's gravity at a point/small region. Gravitational waves are just a perturbation and those emitted by Earth are negligibly weak, but if you had a pair of merging black holes relatively nearby, then indeed, who knows what the result would be?
                                    – Mithrandir24601
                                    Aug 9 at 23:27












                                  up vote
                                  3
                                  down vote










                                  up vote
                                  3
                                  down vote









                                  Gravity As A Wave



                                  Currently we don't know if gravity is a wave or a particle, but I've read it theorized that it's both, similar to photons. If we assume that gravity can exist as a wave, then perhaps a wave that happen to be the inverse of the gravitational wave signature of Earth's gravity well could cancel out that waveform an therefore nullify gravity. I'm not sure if destructive interference would actually apply to gravity waves, but maybe!



                                  I would expect something like that to be temporary or fleeting, but there are plenty of cosmological events that might create gravity waves. It could even be a failed human experiment or intentional attack from an outside force.



                                  Destructive Interference






                                  share|improve this answer












                                  Gravity As A Wave



                                  Currently we don't know if gravity is a wave or a particle, but I've read it theorized that it's both, similar to photons. If we assume that gravity can exist as a wave, then perhaps a wave that happen to be the inverse of the gravitational wave signature of Earth's gravity well could cancel out that waveform an therefore nullify gravity. I'm not sure if destructive interference would actually apply to gravity waves, but maybe!



                                  I would expect something like that to be temporary or fleeting, but there are plenty of cosmological events that might create gravity waves. It could even be a failed human experiment or intentional attack from an outside force.



                                  Destructive Interference







                                  share|improve this answer












                                  share|improve this answer



                                  share|improve this answer










                                  answered Aug 9 at 23:09









                                  Ryan Gibbs

                                  1312




                                  1312







                                  • 3




                                    Welcome to Worldbuilding! Just so you're aware, they have experimentally verified that gravitational waves (i.e. what it looks like you're talking about) do exist. It's a bit more complicated than this but the general idea of putting something that gives an equally strong gravitational field but with the force in the opposite direction would cancel out Earth's gravity at a point/small region. Gravitational waves are just a perturbation and those emitted by Earth are negligibly weak, but if you had a pair of merging black holes relatively nearby, then indeed, who knows what the result would be?
                                    – Mithrandir24601
                                    Aug 9 at 23:27












                                  • 3




                                    Welcome to Worldbuilding! Just so you're aware, they have experimentally verified that gravitational waves (i.e. what it looks like you're talking about) do exist. It's a bit more complicated than this but the general idea of putting something that gives an equally strong gravitational field but with the force in the opposite direction would cancel out Earth's gravity at a point/small region. Gravitational waves are just a perturbation and those emitted by Earth are negligibly weak, but if you had a pair of merging black holes relatively nearby, then indeed, who knows what the result would be?
                                    – Mithrandir24601
                                    Aug 9 at 23:27







                                  3




                                  3




                                  Welcome to Worldbuilding! Just so you're aware, they have experimentally verified that gravitational waves (i.e. what it looks like you're talking about) do exist. It's a bit more complicated than this but the general idea of putting something that gives an equally strong gravitational field but with the force in the opposite direction would cancel out Earth's gravity at a point/small region. Gravitational waves are just a perturbation and those emitted by Earth are negligibly weak, but if you had a pair of merging black holes relatively nearby, then indeed, who knows what the result would be?
                                  – Mithrandir24601
                                  Aug 9 at 23:27




                                  Welcome to Worldbuilding! Just so you're aware, they have experimentally verified that gravitational waves (i.e. what it looks like you're talking about) do exist. It's a bit more complicated than this but the general idea of putting something that gives an equally strong gravitational field but with the force in the opposite direction would cancel out Earth's gravity at a point/small region. Gravitational waves are just a perturbation and those emitted by Earth are negligibly weak, but if you had a pair of merging black holes relatively nearby, then indeed, who knows what the result would be?
                                  – Mithrandir24601
                                  Aug 9 at 23:27










                                  up vote
                                  2
                                  down vote













                                  This wouldn't necessarily reduce gravity, but it would reduce the sensation of gravity.



                                  Make the Earth start Spinning faster for some reason.



                                  Maybe the core's Dynamo got stronger due to (insert handwave here) resulting in the Earth spinning faster due to the Handwave Principle of Handaway.



                                  The centrifugal force would mean that everything on Earth would feel a slight force push them upward, and the net force felt by things on Earth would be weaker.






                                  share|improve this answer




















                                  • This would only affect gravity perpendicular to the axis of rotation. Earth already has different axial and equatorial diameters, not to mention gravities. By the time you get enough spin to counteract gravity entirely, you'd have a lot of shape deformation, not to mention some pretty crazy gravity differences as you change latitude
                                    – Punintended
                                    Aug 9 at 19:36







                                  • 2




                                    @Punintended If you get enough spin to counteract gravity, that would necessarily rip the Earth apart. Yes, increasing the Earth's spin would not result in equal gravity, but it would mean that you might get crazy weather and ocean currents, making the consequences even worse, which is good for the story.
                                    – Totillity
                                    Aug 9 at 19:40










                                  • The planet Mesklin from the old sci-fi classic "Mission of Gravity" was a high-g world spinning fast enough that the gravity was human-survivable (although not comfortable) at the equator.
                                    – Tim B♦
                                    Aug 9 at 20:59














                                  up vote
                                  2
                                  down vote













                                  This wouldn't necessarily reduce gravity, but it would reduce the sensation of gravity.



                                  Make the Earth start Spinning faster for some reason.



                                  Maybe the core's Dynamo got stronger due to (insert handwave here) resulting in the Earth spinning faster due to the Handwave Principle of Handaway.



                                  The centrifugal force would mean that everything on Earth would feel a slight force push them upward, and the net force felt by things on Earth would be weaker.






                                  share|improve this answer




















                                  • This would only affect gravity perpendicular to the axis of rotation. Earth already has different axial and equatorial diameters, not to mention gravities. By the time you get enough spin to counteract gravity entirely, you'd have a lot of shape deformation, not to mention some pretty crazy gravity differences as you change latitude
                                    – Punintended
                                    Aug 9 at 19:36







                                  • 2




                                    @Punintended If you get enough spin to counteract gravity, that would necessarily rip the Earth apart. Yes, increasing the Earth's spin would not result in equal gravity, but it would mean that you might get crazy weather and ocean currents, making the consequences even worse, which is good for the story.
                                    – Totillity
                                    Aug 9 at 19:40










                                  • The planet Mesklin from the old sci-fi classic "Mission of Gravity" was a high-g world spinning fast enough that the gravity was human-survivable (although not comfortable) at the equator.
                                    – Tim B♦
                                    Aug 9 at 20:59












                                  up vote
                                  2
                                  down vote










                                  up vote
                                  2
                                  down vote









                                  This wouldn't necessarily reduce gravity, but it would reduce the sensation of gravity.



                                  Make the Earth start Spinning faster for some reason.



                                  Maybe the core's Dynamo got stronger due to (insert handwave here) resulting in the Earth spinning faster due to the Handwave Principle of Handaway.



                                  The centrifugal force would mean that everything on Earth would feel a slight force push them upward, and the net force felt by things on Earth would be weaker.






                                  share|improve this answer












                                  This wouldn't necessarily reduce gravity, but it would reduce the sensation of gravity.



                                  Make the Earth start Spinning faster for some reason.



                                  Maybe the core's Dynamo got stronger due to (insert handwave here) resulting in the Earth spinning faster due to the Handwave Principle of Handaway.



                                  The centrifugal force would mean that everything on Earth would feel a slight force push them upward, and the net force felt by things on Earth would be weaker.







                                  share|improve this answer












                                  share|improve this answer



                                  share|improve this answer










                                  answered Aug 9 at 19:21









                                  Totillity

                                  2466




                                  2466











                                  • This would only affect gravity perpendicular to the axis of rotation. Earth already has different axial and equatorial diameters, not to mention gravities. By the time you get enough spin to counteract gravity entirely, you'd have a lot of shape deformation, not to mention some pretty crazy gravity differences as you change latitude
                                    – Punintended
                                    Aug 9 at 19:36







                                  • 2




                                    @Punintended If you get enough spin to counteract gravity, that would necessarily rip the Earth apart. Yes, increasing the Earth's spin would not result in equal gravity, but it would mean that you might get crazy weather and ocean currents, making the consequences even worse, which is good for the story.
                                    – Totillity
                                    Aug 9 at 19:40










                                  • The planet Mesklin from the old sci-fi classic "Mission of Gravity" was a high-g world spinning fast enough that the gravity was human-survivable (although not comfortable) at the equator.
                                    – Tim B♦
                                    Aug 9 at 20:59
















                                  • This would only affect gravity perpendicular to the axis of rotation. Earth already has different axial and equatorial diameters, not to mention gravities. By the time you get enough spin to counteract gravity entirely, you'd have a lot of shape deformation, not to mention some pretty crazy gravity differences as you change latitude
                                    – Punintended
                                    Aug 9 at 19:36







                                  • 2




                                    @Punintended If you get enough spin to counteract gravity, that would necessarily rip the Earth apart. Yes, increasing the Earth's spin would not result in equal gravity, but it would mean that you might get crazy weather and ocean currents, making the consequences even worse, which is good for the story.
                                    – Totillity
                                    Aug 9 at 19:40










                                  • The planet Mesklin from the old sci-fi classic "Mission of Gravity" was a high-g world spinning fast enough that the gravity was human-survivable (although not comfortable) at the equator.
                                    – Tim B♦
                                    Aug 9 at 20:59















                                  This would only affect gravity perpendicular to the axis of rotation. Earth already has different axial and equatorial diameters, not to mention gravities. By the time you get enough spin to counteract gravity entirely, you'd have a lot of shape deformation, not to mention some pretty crazy gravity differences as you change latitude
                                  – Punintended
                                  Aug 9 at 19:36





                                  This would only affect gravity perpendicular to the axis of rotation. Earth already has different axial and equatorial diameters, not to mention gravities. By the time you get enough spin to counteract gravity entirely, you'd have a lot of shape deformation, not to mention some pretty crazy gravity differences as you change latitude
                                  – Punintended
                                  Aug 9 at 19:36





                                  2




                                  2




                                  @Punintended If you get enough spin to counteract gravity, that would necessarily rip the Earth apart. Yes, increasing the Earth's spin would not result in equal gravity, but it would mean that you might get crazy weather and ocean currents, making the consequences even worse, which is good for the story.
                                  – Totillity
                                  Aug 9 at 19:40




                                  @Punintended If you get enough spin to counteract gravity, that would necessarily rip the Earth apart. Yes, increasing the Earth's spin would not result in equal gravity, but it would mean that you might get crazy weather and ocean currents, making the consequences even worse, which is good for the story.
                                  – Totillity
                                  Aug 9 at 19:40












                                  The planet Mesklin from the old sci-fi classic "Mission of Gravity" was a high-g world spinning fast enough that the gravity was human-survivable (although not comfortable) at the equator.
                                  – Tim B♦
                                  Aug 9 at 20:59




                                  The planet Mesklin from the old sci-fi classic "Mission of Gravity" was a high-g world spinning fast enough that the gravity was human-survivable (although not comfortable) at the equator.
                                  – Tim B♦
                                  Aug 9 at 20:59










                                  up vote
                                  2
                                  down vote













                                  While you can't turn off gravity without getting rid of mass, it's possible to mess it up by counteracting it with another invisible force pointing in a different direction. For instance, when you're in a car that takes a sharp turn, you can feel being pushed outward. This is the centrifugal force, and it can even dominate over gravity when the turn is sharp enough, in which case the car flips over.



                                  How is this relevant to your case? See, we all experience a centrifugal force because of the earth's spin. A fraction of this force counteracts gravity (which always points to the core), however the effect is tiny because the earth spins slowly and has a huge radius: The closer you are at the equator, the stronger the effect. So, if you want to mess up gravity on earth, you can do so by making it spin faster.



                                  There's actually a formula you can use to calculate the centrifugal force, and compare it to gravity. Without getting into details, you can do some back-of-the-envelope calculations to determine that earth's gravity would start getting messed up by centrifugal forces if it were to spin ~10 times faster. That is, if the duration of a day (sunrise to sunset) were ~1hr long.



                                  How exactly would earth's gravity get messed up? In short, people at the poles would be unaffected, while those at the equator would basically start floating around (along with the entirety of the atmosphere). Things wouldn't be flung into space, though, because of inertia. People in between would feel lighter (reduced gravity), as well as experience a mysterious pull toward the equator (i.e., southward in the northern hemisphere, and northward in the southern hemisphere).



                                  In the above apocalyptic scenario, you can imagine that, as the earth spins faster and faster, a certain priviledged few would travel hurriedly toward the poles as the rest of humanity (as well as the atmosphere) gets pulled increasingly strongly toward the equator. It would be a very bizarre situation indeed, as what we all perceive as "down" would no longer feel like "down" anymore; as the earth's spins increases, "down" would increasingly be directed toward the horizon and in the direction of the equator. Not to mention you'd feel lighter and lighter, which, under "normal" conditions, is quite a pleasurable experience.



                                  There's a catch, of course: How do you actually spin the earth up in a fairly reasonable fashion as to make for a dramatic human story that's actually based on well-established scientific principles? One way I could think of, is reducing the earth's moment of inertia; i.e., changing its internal mass distribution. The more concentrated a body's mass is distributed along the rotation axis, the faster it rotates (conservation of angular momentum). So, without changing anything on the surface where the human drama unfolds, you need a mechanism that can plausibly cause the internal mass to get more concentrated toward the core.



                                  By playing around with some numbers, it turns out that you need to cram ~99% of the earth's mass right at the core to make the earth spin faster by a factor of ~10. The only mechanism I can think of that could possibly cause such a huge re-distribution of mass, is a swarm of black holes orbiting around each other in the vicinity of the earth's core. Note that these would have to be tiny, since an earth-mass black hole is ~1cm in size and you obviously don't want to add any mass to the earth, since that would mess up "natural" gravity.



                                  In any case, I didn't investigate the dynamics of a swarm of black holes; i.e., I don't know how quickly they could gobble up a good fraction of the earth's mass as to make the spin-up effect noticeable within a reasonable human timeframe (e.g., years of months). I can't even comment on the stability of such a hypothetical system, since I didn't do any pertinent calculations.






                                  share|improve this answer




















                                  • Just read an article about what would happen if the earth spun faster popsci.com/earth-spin-faster#page-4 !
                                    – Jaco Briers
                                    Aug 10 at 18:00














                                  up vote
                                  2
                                  down vote













                                  While you can't turn off gravity without getting rid of mass, it's possible to mess it up by counteracting it with another invisible force pointing in a different direction. For instance, when you're in a car that takes a sharp turn, you can feel being pushed outward. This is the centrifugal force, and it can even dominate over gravity when the turn is sharp enough, in which case the car flips over.



                                  How is this relevant to your case? See, we all experience a centrifugal force because of the earth's spin. A fraction of this force counteracts gravity (which always points to the core), however the effect is tiny because the earth spins slowly and has a huge radius: The closer you are at the equator, the stronger the effect. So, if you want to mess up gravity on earth, you can do so by making it spin faster.



                                  There's actually a formula you can use to calculate the centrifugal force, and compare it to gravity. Without getting into details, you can do some back-of-the-envelope calculations to determine that earth's gravity would start getting messed up by centrifugal forces if it were to spin ~10 times faster. That is, if the duration of a day (sunrise to sunset) were ~1hr long.



                                  How exactly would earth's gravity get messed up? In short, people at the poles would be unaffected, while those at the equator would basically start floating around (along with the entirety of the atmosphere). Things wouldn't be flung into space, though, because of inertia. People in between would feel lighter (reduced gravity), as well as experience a mysterious pull toward the equator (i.e., southward in the northern hemisphere, and northward in the southern hemisphere).



                                  In the above apocalyptic scenario, you can imagine that, as the earth spins faster and faster, a certain priviledged few would travel hurriedly toward the poles as the rest of humanity (as well as the atmosphere) gets pulled increasingly strongly toward the equator. It would be a very bizarre situation indeed, as what we all perceive as "down" would no longer feel like "down" anymore; as the earth's spins increases, "down" would increasingly be directed toward the horizon and in the direction of the equator. Not to mention you'd feel lighter and lighter, which, under "normal" conditions, is quite a pleasurable experience.



                                  There's a catch, of course: How do you actually spin the earth up in a fairly reasonable fashion as to make for a dramatic human story that's actually based on well-established scientific principles? One way I could think of, is reducing the earth's moment of inertia; i.e., changing its internal mass distribution. The more concentrated a body's mass is distributed along the rotation axis, the faster it rotates (conservation of angular momentum). So, without changing anything on the surface where the human drama unfolds, you need a mechanism that can plausibly cause the internal mass to get more concentrated toward the core.



                                  By playing around with some numbers, it turns out that you need to cram ~99% of the earth's mass right at the core to make the earth spin faster by a factor of ~10. The only mechanism I can think of that could possibly cause such a huge re-distribution of mass, is a swarm of black holes orbiting around each other in the vicinity of the earth's core. Note that these would have to be tiny, since an earth-mass black hole is ~1cm in size and you obviously don't want to add any mass to the earth, since that would mess up "natural" gravity.



                                  In any case, I didn't investigate the dynamics of a swarm of black holes; i.e., I don't know how quickly they could gobble up a good fraction of the earth's mass as to make the spin-up effect noticeable within a reasonable human timeframe (e.g., years of months). I can't even comment on the stability of such a hypothetical system, since I didn't do any pertinent calculations.






                                  share|improve this answer




















                                  • Just read an article about what would happen if the earth spun faster popsci.com/earth-spin-faster#page-4 !
                                    – Jaco Briers
                                    Aug 10 at 18:00












                                  up vote
                                  2
                                  down vote










                                  up vote
                                  2
                                  down vote









                                  While you can't turn off gravity without getting rid of mass, it's possible to mess it up by counteracting it with another invisible force pointing in a different direction. For instance, when you're in a car that takes a sharp turn, you can feel being pushed outward. This is the centrifugal force, and it can even dominate over gravity when the turn is sharp enough, in which case the car flips over.



                                  How is this relevant to your case? See, we all experience a centrifugal force because of the earth's spin. A fraction of this force counteracts gravity (which always points to the core), however the effect is tiny because the earth spins slowly and has a huge radius: The closer you are at the equator, the stronger the effect. So, if you want to mess up gravity on earth, you can do so by making it spin faster.



                                  There's actually a formula you can use to calculate the centrifugal force, and compare it to gravity. Without getting into details, you can do some back-of-the-envelope calculations to determine that earth's gravity would start getting messed up by centrifugal forces if it were to spin ~10 times faster. That is, if the duration of a day (sunrise to sunset) were ~1hr long.



                                  How exactly would earth's gravity get messed up? In short, people at the poles would be unaffected, while those at the equator would basically start floating around (along with the entirety of the atmosphere). Things wouldn't be flung into space, though, because of inertia. People in between would feel lighter (reduced gravity), as well as experience a mysterious pull toward the equator (i.e., southward in the northern hemisphere, and northward in the southern hemisphere).



                                  In the above apocalyptic scenario, you can imagine that, as the earth spins faster and faster, a certain priviledged few would travel hurriedly toward the poles as the rest of humanity (as well as the atmosphere) gets pulled increasingly strongly toward the equator. It would be a very bizarre situation indeed, as what we all perceive as "down" would no longer feel like "down" anymore; as the earth's spins increases, "down" would increasingly be directed toward the horizon and in the direction of the equator. Not to mention you'd feel lighter and lighter, which, under "normal" conditions, is quite a pleasurable experience.



                                  There's a catch, of course: How do you actually spin the earth up in a fairly reasonable fashion as to make for a dramatic human story that's actually based on well-established scientific principles? One way I could think of, is reducing the earth's moment of inertia; i.e., changing its internal mass distribution. The more concentrated a body's mass is distributed along the rotation axis, the faster it rotates (conservation of angular momentum). So, without changing anything on the surface where the human drama unfolds, you need a mechanism that can plausibly cause the internal mass to get more concentrated toward the core.



                                  By playing around with some numbers, it turns out that you need to cram ~99% of the earth's mass right at the core to make the earth spin faster by a factor of ~10. The only mechanism I can think of that could possibly cause such a huge re-distribution of mass, is a swarm of black holes orbiting around each other in the vicinity of the earth's core. Note that these would have to be tiny, since an earth-mass black hole is ~1cm in size and you obviously don't want to add any mass to the earth, since that would mess up "natural" gravity.



                                  In any case, I didn't investigate the dynamics of a swarm of black holes; i.e., I don't know how quickly they could gobble up a good fraction of the earth's mass as to make the spin-up effect noticeable within a reasonable human timeframe (e.g., years of months). I can't even comment on the stability of such a hypothetical system, since I didn't do any pertinent calculations.






                                  share|improve this answer












                                  While you can't turn off gravity without getting rid of mass, it's possible to mess it up by counteracting it with another invisible force pointing in a different direction. For instance, when you're in a car that takes a sharp turn, you can feel being pushed outward. This is the centrifugal force, and it can even dominate over gravity when the turn is sharp enough, in which case the car flips over.



                                  How is this relevant to your case? See, we all experience a centrifugal force because of the earth's spin. A fraction of this force counteracts gravity (which always points to the core), however the effect is tiny because the earth spins slowly and has a huge radius: The closer you are at the equator, the stronger the effect. So, if you want to mess up gravity on earth, you can do so by making it spin faster.



                                  There's actually a formula you can use to calculate the centrifugal force, and compare it to gravity. Without getting into details, you can do some back-of-the-envelope calculations to determine that earth's gravity would start getting messed up by centrifugal forces if it were to spin ~10 times faster. That is, if the duration of a day (sunrise to sunset) were ~1hr long.



                                  How exactly would earth's gravity get messed up? In short, people at the poles would be unaffected, while those at the equator would basically start floating around (along with the entirety of the atmosphere). Things wouldn't be flung into space, though, because of inertia. People in between would feel lighter (reduced gravity), as well as experience a mysterious pull toward the equator (i.e., southward in the northern hemisphere, and northward in the southern hemisphere).



                                  In the above apocalyptic scenario, you can imagine that, as the earth spins faster and faster, a certain priviledged few would travel hurriedly toward the poles as the rest of humanity (as well as the atmosphere) gets pulled increasingly strongly toward the equator. It would be a very bizarre situation indeed, as what we all perceive as "down" would no longer feel like "down" anymore; as the earth's spins increases, "down" would increasingly be directed toward the horizon and in the direction of the equator. Not to mention you'd feel lighter and lighter, which, under "normal" conditions, is quite a pleasurable experience.



                                  There's a catch, of course: How do you actually spin the earth up in a fairly reasonable fashion as to make for a dramatic human story that's actually based on well-established scientific principles? One way I could think of, is reducing the earth's moment of inertia; i.e., changing its internal mass distribution. The more concentrated a body's mass is distributed along the rotation axis, the faster it rotates (conservation of angular momentum). So, without changing anything on the surface where the human drama unfolds, you need a mechanism that can plausibly cause the internal mass to get more concentrated toward the core.



                                  By playing around with some numbers, it turns out that you need to cram ~99% of the earth's mass right at the core to make the earth spin faster by a factor of ~10. The only mechanism I can think of that could possibly cause such a huge re-distribution of mass, is a swarm of black holes orbiting around each other in the vicinity of the earth's core. Note that these would have to be tiny, since an earth-mass black hole is ~1cm in size and you obviously don't want to add any mass to the earth, since that would mess up "natural" gravity.



                                  In any case, I didn't investigate the dynamics of a swarm of black holes; i.e., I don't know how quickly they could gobble up a good fraction of the earth's mass as to make the spin-up effect noticeable within a reasonable human timeframe (e.g., years of months). I can't even comment on the stability of such a hypothetical system, since I didn't do any pertinent calculations.







                                  share|improve this answer












                                  share|improve this answer



                                  share|improve this answer










                                  answered Aug 10 at 13:49









                                  user54068

                                  211




                                  211











                                  • Just read an article about what would happen if the earth spun faster popsci.com/earth-spin-faster#page-4 !
                                    – Jaco Briers
                                    Aug 10 at 18:00
















                                  • Just read an article about what would happen if the earth spun faster popsci.com/earth-spin-faster#page-4 !
                                    – Jaco Briers
                                    Aug 10 at 18:00















                                  Just read an article about what would happen if the earth spun faster popsci.com/earth-spin-faster#page-4 !
                                  – Jaco Briers
                                  Aug 10 at 18:00




                                  Just read an article about what would happen if the earth spun faster popsci.com/earth-spin-faster#page-4 !
                                  – Jaco Briers
                                  Aug 10 at 18:00










                                  up vote
                                  1
                                  down vote













                                  I would suggest something in the lines of what happened at the end of Melancholia: a second large mass comes very close (and for some reason stays put at the required distance to make the story interesting), pretty much deforming gravity on Earth. If you need it to be transparent, non-spherically shaped, anything works really. Not every piece of debris (or alien planet-ship) is perfectly spherical.



                                  For more discussion on the plausibility and effects of such an event, see these network questions:



                                  https://scifi.stackexchange.com/questions/19068/is-the-melancholia-dance-of-death-possible



                                  https://physics.stackexchange.com/questions/15083/is-melancholias-orbit-impossible



                                  Note I am suggesting some articifial means to keep the large second object in place, enough to disturb pretty much everything gravity related, without breaking any of the underlying physics.
                                  Also be aware that normal gravity is the single most important thing keeping our atmosphere together.






                                  share|improve this answer
























                                    up vote
                                    1
                                    down vote













                                    I would suggest something in the lines of what happened at the end of Melancholia: a second large mass comes very close (and for some reason stays put at the required distance to make the story interesting), pretty much deforming gravity on Earth. If you need it to be transparent, non-spherically shaped, anything works really. Not every piece of debris (or alien planet-ship) is perfectly spherical.



                                    For more discussion on the plausibility and effects of such an event, see these network questions:



                                    https://scifi.stackexchange.com/questions/19068/is-the-melancholia-dance-of-death-possible



                                    https://physics.stackexchange.com/questions/15083/is-melancholias-orbit-impossible



                                    Note I am suggesting some articifial means to keep the large second object in place, enough to disturb pretty much everything gravity related, without breaking any of the underlying physics.
                                    Also be aware that normal gravity is the single most important thing keeping our atmosphere together.






                                    share|improve this answer






















                                      up vote
                                      1
                                      down vote










                                      up vote
                                      1
                                      down vote









                                      I would suggest something in the lines of what happened at the end of Melancholia: a second large mass comes very close (and for some reason stays put at the required distance to make the story interesting), pretty much deforming gravity on Earth. If you need it to be transparent, non-spherically shaped, anything works really. Not every piece of debris (or alien planet-ship) is perfectly spherical.



                                      For more discussion on the plausibility and effects of such an event, see these network questions:



                                      https://scifi.stackexchange.com/questions/19068/is-the-melancholia-dance-of-death-possible



                                      https://physics.stackexchange.com/questions/15083/is-melancholias-orbit-impossible



                                      Note I am suggesting some articifial means to keep the large second object in place, enough to disturb pretty much everything gravity related, without breaking any of the underlying physics.
                                      Also be aware that normal gravity is the single most important thing keeping our atmosphere together.






                                      share|improve this answer












                                      I would suggest something in the lines of what happened at the end of Melancholia: a second large mass comes very close (and for some reason stays put at the required distance to make the story interesting), pretty much deforming gravity on Earth. If you need it to be transparent, non-spherically shaped, anything works really. Not every piece of debris (or alien planet-ship) is perfectly spherical.



                                      For more discussion on the plausibility and effects of such an event, see these network questions:



                                      https://scifi.stackexchange.com/questions/19068/is-the-melancholia-dance-of-death-possible



                                      https://physics.stackexchange.com/questions/15083/is-melancholias-orbit-impossible



                                      Note I am suggesting some articifial means to keep the large second object in place, enough to disturb pretty much everything gravity related, without breaking any of the underlying physics.
                                      Also be aware that normal gravity is the single most important thing keeping our atmosphere together.







                                      share|improve this answer












                                      share|improve this answer



                                      share|improve this answer










                                      answered Aug 10 at 7:31









                                      rubenvb

                                      1113




                                      1113




















                                          up vote
                                          1
                                          down vote













                                          Thinking about this, I realised that the shell theorem is your friend, and would need to stretch reality a lot less than some other solutions. I may have to backtrack a bit to explain this, as it is indeed based on hard science.



                                          The problem in the question is that as far as we know, gravity is basically, positive only - there isn't negative (anti) gravity, or gravity shields. So the classic solutions have to centre around either mass reduction, or increasingly unlikely/pseudoscientific explanations with implausible implications.



                                          But there is another way gravity can fail. Let's start with the shell theorem. This is a mathematical/physics principle, that if you are anywhere inside a hollow symmetrical spherical shell, of any thickness, you don't experience any net gravity from it - it all cancels out. (Less mass but closer on one side, more mass but further away on the other, and they always exactly match).



                                          But suppose mass appears above you, that isn't part of a symmetrical shell. Now, you will experience some reduction in earths gravity locally, because of the upward pull. Just not very much.



                                          You'll have to check this out, but I guess this could work, especially given that the question allows for limited scientific plausibility:



                                          1. a research satellite is launched to perform some kind of extreme research in the vacuum of space, in some suitable orbit just outside earth's atmosphere.


                                          2. Unknown to those running it, and for [handwave] reasons, the satellite is acting as some kind of dense matter attractor - maybe it warps space or leaves behind some particle, or locally perturbs the vacuum state.


                                          3. The end result is that a series of mini-black holes appear in its wake, conveniently growing and of a size sufficient to influence earths surface gravity below.


                                          4. The satellite of course fails, after one or 2 orbits, but the mass concentrations continue to grow. They don't form a symmetrical shell, but a linear trail. They inherit the velocity of the satellite, so they don't immediately just fall inward to earth?


                                          5. The earth isn't affected much in its orbit - at least not immediately - but if figures out carefully, maybe there would be a local/regional impact on surface gravity, with scope for it to get worse, and other side effects, without having to "break" physics or cause immediate planetary breakup.....? It would have a more severe effect on high altitude objects, as well, which might be useful in a story.






                                          share|improve this answer
























                                            up vote
                                            1
                                            down vote













                                            Thinking about this, I realised that the shell theorem is your friend, and would need to stretch reality a lot less than some other solutions. I may have to backtrack a bit to explain this, as it is indeed based on hard science.



                                            The problem in the question is that as far as we know, gravity is basically, positive only - there isn't negative (anti) gravity, or gravity shields. So the classic solutions have to centre around either mass reduction, or increasingly unlikely/pseudoscientific explanations with implausible implications.



                                            But there is another way gravity can fail. Let's start with the shell theorem. This is a mathematical/physics principle, that if you are anywhere inside a hollow symmetrical spherical shell, of any thickness, you don't experience any net gravity from it - it all cancels out. (Less mass but closer on one side, more mass but further away on the other, and they always exactly match).



                                            But suppose mass appears above you, that isn't part of a symmetrical shell. Now, you will experience some reduction in earths gravity locally, because of the upward pull. Just not very much.



                                            You'll have to check this out, but I guess this could work, especially given that the question allows for limited scientific plausibility:



                                            1. a research satellite is launched to perform some kind of extreme research in the vacuum of space, in some suitable orbit just outside earth's atmosphere.


                                            2. Unknown to those running it, and for [handwave] reasons, the satellite is acting as some kind of dense matter attractor - maybe it warps space or leaves behind some particle, or locally perturbs the vacuum state.


                                            3. The end result is that a series of mini-black holes appear in its wake, conveniently growing and of a size sufficient to influence earths surface gravity below.


                                            4. The satellite of course fails, after one or 2 orbits, but the mass concentrations continue to grow. They don't form a symmetrical shell, but a linear trail. They inherit the velocity of the satellite, so they don't immediately just fall inward to earth?


                                            5. The earth isn't affected much in its orbit - at least not immediately - but if figures out carefully, maybe there would be a local/regional impact on surface gravity, with scope for it to get worse, and other side effects, without having to "break" physics or cause immediate planetary breakup.....? It would have a more severe effect on high altitude objects, as well, which might be useful in a story.






                                            share|improve this answer






















                                              up vote
                                              1
                                              down vote










                                              up vote
                                              1
                                              down vote









                                              Thinking about this, I realised that the shell theorem is your friend, and would need to stretch reality a lot less than some other solutions. I may have to backtrack a bit to explain this, as it is indeed based on hard science.



                                              The problem in the question is that as far as we know, gravity is basically, positive only - there isn't negative (anti) gravity, or gravity shields. So the classic solutions have to centre around either mass reduction, or increasingly unlikely/pseudoscientific explanations with implausible implications.



                                              But there is another way gravity can fail. Let's start with the shell theorem. This is a mathematical/physics principle, that if you are anywhere inside a hollow symmetrical spherical shell, of any thickness, you don't experience any net gravity from it - it all cancels out. (Less mass but closer on one side, more mass but further away on the other, and they always exactly match).



                                              But suppose mass appears above you, that isn't part of a symmetrical shell. Now, you will experience some reduction in earths gravity locally, because of the upward pull. Just not very much.



                                              You'll have to check this out, but I guess this could work, especially given that the question allows for limited scientific plausibility:



                                              1. a research satellite is launched to perform some kind of extreme research in the vacuum of space, in some suitable orbit just outside earth's atmosphere.


                                              2. Unknown to those running it, and for [handwave] reasons, the satellite is acting as some kind of dense matter attractor - maybe it warps space or leaves behind some particle, or locally perturbs the vacuum state.


                                              3. The end result is that a series of mini-black holes appear in its wake, conveniently growing and of a size sufficient to influence earths surface gravity below.


                                              4. The satellite of course fails, after one or 2 orbits, but the mass concentrations continue to grow. They don't form a symmetrical shell, but a linear trail. They inherit the velocity of the satellite, so they don't immediately just fall inward to earth?


                                              5. The earth isn't affected much in its orbit - at least not immediately - but if figures out carefully, maybe there would be a local/regional impact on surface gravity, with scope for it to get worse, and other side effects, without having to "break" physics or cause immediate planetary breakup.....? It would have a more severe effect on high altitude objects, as well, which might be useful in a story.






                                              share|improve this answer












                                              Thinking about this, I realised that the shell theorem is your friend, and would need to stretch reality a lot less than some other solutions. I may have to backtrack a bit to explain this, as it is indeed based on hard science.



                                              The problem in the question is that as far as we know, gravity is basically, positive only - there isn't negative (anti) gravity, or gravity shields. So the classic solutions have to centre around either mass reduction, or increasingly unlikely/pseudoscientific explanations with implausible implications.



                                              But there is another way gravity can fail. Let's start with the shell theorem. This is a mathematical/physics principle, that if you are anywhere inside a hollow symmetrical spherical shell, of any thickness, you don't experience any net gravity from it - it all cancels out. (Less mass but closer on one side, more mass but further away on the other, and they always exactly match).



                                              But suppose mass appears above you, that isn't part of a symmetrical shell. Now, you will experience some reduction in earths gravity locally, because of the upward pull. Just not very much.



                                              You'll have to check this out, but I guess this could work, especially given that the question allows for limited scientific plausibility:



                                              1. a research satellite is launched to perform some kind of extreme research in the vacuum of space, in some suitable orbit just outside earth's atmosphere.


                                              2. Unknown to those running it, and for [handwave] reasons, the satellite is acting as some kind of dense matter attractor - maybe it warps space or leaves behind some particle, or locally perturbs the vacuum state.


                                              3. The end result is that a series of mini-black holes appear in its wake, conveniently growing and of a size sufficient to influence earths surface gravity below.


                                              4. The satellite of course fails, after one or 2 orbits, but the mass concentrations continue to grow. They don't form a symmetrical shell, but a linear trail. They inherit the velocity of the satellite, so they don't immediately just fall inward to earth?


                                              5. The earth isn't affected much in its orbit - at least not immediately - but if figures out carefully, maybe there would be a local/regional impact on surface gravity, with scope for it to get worse, and other side effects, without having to "break" physics or cause immediate planetary breakup.....? It would have a more severe effect on high altitude objects, as well, which might be useful in a story.







                                              share|improve this answer












                                              share|improve this answer



                                              share|improve this answer










                                              answered Aug 11 at 20:24









                                              Stilez

                                              2,57469




                                              2,57469




















                                                  up vote
                                                  1
                                                  down vote













                                                  Negative mass



                                                  • We had taken for granted for ages that gravity is only attractive. But it turned out that it wasn't.


                                                  • While trying to create chambers of absolute vacuum, it turned out that speed of effect of gravitation is related to the absolute amount of gravitational entropy ( how much of the available mass of the universe can be present at a given location).


                                                  • By creating a perfect vacuum scientists were able to stop the effects of gravity within a region.


                                                  • What was originally understood as vacuum wasn't really vacuum, but a region of space of negative mass which in the presence of other positive behaved like vacuum.



                                                  • Immediately new research begun and more and more people start researching on this. Meanwhile it turns out that by creating more and more perfect vacuums, we are creating more regions of negative mass.



                                                    • This negative mass is termed as Anti Dark Matter for it only interacts through gravitational forces and is repulsive.


                                                  • Termination of a vacuum doesn't always ensure that all the positive mass covers up the negative.



                                                  • As the gravitational interactions are very weak it was only noticed many more years later that the experiments which were run had caused a small leakage of negative mass that had somehow made it to the near the earth's outer core where it nicely found and equilibrium and settled.



                                                    • As Earth passes through various regions of space, overtime this negative mass at the core had been collecting more and more of negative mass, and it is growing exponentially.


                                                    • It is cancelling out the effects of the earth's core and mantle by overlapping and creating regions of 'vacuum'







                                                  share|improve this answer
























                                                    up vote
                                                    1
                                                    down vote













                                                    Negative mass



                                                    • We had taken for granted for ages that gravity is only attractive. But it turned out that it wasn't.


                                                    • While trying to create chambers of absolute vacuum, it turned out that speed of effect of gravitation is related to the absolute amount of gravitational entropy ( how much of the available mass of the universe can be present at a given location).


                                                    • By creating a perfect vacuum scientists were able to stop the effects of gravity within a region.


                                                    • What was originally understood as vacuum wasn't really vacuum, but a region of space of negative mass which in the presence of other positive behaved like vacuum.



                                                    • Immediately new research begun and more and more people start researching on this. Meanwhile it turns out that by creating more and more perfect vacuums, we are creating more regions of negative mass.



                                                      • This negative mass is termed as Anti Dark Matter for it only interacts through gravitational forces and is repulsive.


                                                    • Termination of a vacuum doesn't always ensure that all the positive mass covers up the negative.



                                                    • As the gravitational interactions are very weak it was only noticed many more years later that the experiments which were run had caused a small leakage of negative mass that had somehow made it to the near the earth's outer core where it nicely found and equilibrium and settled.



                                                      • As Earth passes through various regions of space, overtime this negative mass at the core had been collecting more and more of negative mass, and it is growing exponentially.


                                                      • It is cancelling out the effects of the earth's core and mantle by overlapping and creating regions of 'vacuum'







                                                    share|improve this answer






















                                                      up vote
                                                      1
                                                      down vote










                                                      up vote
                                                      1
                                                      down vote









                                                      Negative mass



                                                      • We had taken for granted for ages that gravity is only attractive. But it turned out that it wasn't.


                                                      • While trying to create chambers of absolute vacuum, it turned out that speed of effect of gravitation is related to the absolute amount of gravitational entropy ( how much of the available mass of the universe can be present at a given location).


                                                      • By creating a perfect vacuum scientists were able to stop the effects of gravity within a region.


                                                      • What was originally understood as vacuum wasn't really vacuum, but a region of space of negative mass which in the presence of other positive behaved like vacuum.



                                                      • Immediately new research begun and more and more people start researching on this. Meanwhile it turns out that by creating more and more perfect vacuums, we are creating more regions of negative mass.



                                                        • This negative mass is termed as Anti Dark Matter for it only interacts through gravitational forces and is repulsive.


                                                      • Termination of a vacuum doesn't always ensure that all the positive mass covers up the negative.



                                                      • As the gravitational interactions are very weak it was only noticed many more years later that the experiments which were run had caused a small leakage of negative mass that had somehow made it to the near the earth's outer core where it nicely found and equilibrium and settled.



                                                        • As Earth passes through various regions of space, overtime this negative mass at the core had been collecting more and more of negative mass, and it is growing exponentially.


                                                        • It is cancelling out the effects of the earth's core and mantle by overlapping and creating regions of 'vacuum'







                                                      share|improve this answer












                                                      Negative mass



                                                      • We had taken for granted for ages that gravity is only attractive. But it turned out that it wasn't.


                                                      • While trying to create chambers of absolute vacuum, it turned out that speed of effect of gravitation is related to the absolute amount of gravitational entropy ( how much of the available mass of the universe can be present at a given location).


                                                      • By creating a perfect vacuum scientists were able to stop the effects of gravity within a region.


                                                      • What was originally understood as vacuum wasn't really vacuum, but a region of space of negative mass which in the presence of other positive behaved like vacuum.



                                                      • Immediately new research begun and more and more people start researching on this. Meanwhile it turns out that by creating more and more perfect vacuums, we are creating more regions of negative mass.



                                                        • This negative mass is termed as Anti Dark Matter for it only interacts through gravitational forces and is repulsive.


                                                      • Termination of a vacuum doesn't always ensure that all the positive mass covers up the negative.



                                                      • As the gravitational interactions are very weak it was only noticed many more years later that the experiments which were run had caused a small leakage of negative mass that had somehow made it to the near the earth's outer core where it nicely found and equilibrium and settled.



                                                        • As Earth passes through various regions of space, overtime this negative mass at the core had been collecting more and more of negative mass, and it is growing exponentially.


                                                        • It is cancelling out the effects of the earth's core and mantle by overlapping and creating regions of 'vacuum'








                                                      share|improve this answer












                                                      share|improve this answer



                                                      share|improve this answer










                                                      answered Aug 12 at 18:02









                                                      Debaditya

                                                      913




                                                      913




















                                                          up vote
                                                          0
                                                          down vote













                                                          Gravity is caused by bending of space time (IRL).



                                                          A big corporation has worked out a way of flattening it artificially - effectively "antigravity", and sells it as a product.






                                                          share|improve this answer
























                                                            up vote
                                                            0
                                                            down vote













                                                            Gravity is caused by bending of space time (IRL).



                                                            A big corporation has worked out a way of flattening it artificially - effectively "antigravity", and sells it as a product.






                                                            share|improve this answer






















                                                              up vote
                                                              0
                                                              down vote










                                                              up vote
                                                              0
                                                              down vote









                                                              Gravity is caused by bending of space time (IRL).



                                                              A big corporation has worked out a way of flattening it artificially - effectively "antigravity", and sells it as a product.






                                                              share|improve this answer












                                                              Gravity is caused by bending of space time (IRL).



                                                              A big corporation has worked out a way of flattening it artificially - effectively "antigravity", and sells it as a product.







                                                              share|improve this answer












                                                              share|improve this answer



                                                              share|improve this answer










                                                              answered Aug 10 at 2:24









                                                              AMADANON Inc.

                                                              1,07137




                                                              1,07137




















                                                                  up vote
                                                                  0
                                                                  down vote













                                                                  The changing/failing gravity is a floating point error in the universe simulation.



                                                                  Many existing simulations of varying complexity use floating point numbers for their calculations. For example, the Unity game engine uses a floating point number for gravity strength. In this case, the floating point number would be the lifetime or duration of the gravity effect. It could even be tracking the lifetime of the universe, and the gravity is just a side effect.






                                                                  share|improve this answer




















                                                                  • Welcome to Worldbuilding, Jarod S! If you have a moment, please take the tour and visit the help center to learn more about the site. You may also find Worldbuilding Meta and The Sandbox (both of which require 5 rep to post on) useful. Here is a meta post on the culture and style of Worldbuilding.SE, just to help you understand our scope and methods, and how we do things here. Have fun!
                                                                    – Gryphon
                                                                    Aug 10 at 17:13














                                                                  up vote
                                                                  0
                                                                  down vote













                                                                  The changing/failing gravity is a floating point error in the universe simulation.



                                                                  Many existing simulations of varying complexity use floating point numbers for their calculations. For example, the Unity game engine uses a floating point number for gravity strength. In this case, the floating point number would be the lifetime or duration of the gravity effect. It could even be tracking the lifetime of the universe, and the gravity is just a side effect.






                                                                  share|improve this answer




















                                                                  • Welcome to Worldbuilding, Jarod S! If you have a moment, please take the tour and visit the help center to learn more about the site. You may also find Worldbuilding Meta and The Sandbox (both of which require 5 rep to post on) useful. Here is a meta post on the culture and style of Worldbuilding.SE, just to help you understand our scope and methods, and how we do things here. Have fun!
                                                                    – Gryphon
                                                                    Aug 10 at 17:13












                                                                  up vote
                                                                  0
                                                                  down vote










                                                                  up vote
                                                                  0
                                                                  down vote









                                                                  The changing/failing gravity is a floating point error in the universe simulation.



                                                                  Many existing simulations of varying complexity use floating point numbers for their calculations. For example, the Unity game engine uses a floating point number for gravity strength. In this case, the floating point number would be the lifetime or duration of the gravity effect. It could even be tracking the lifetime of the universe, and the gravity is just a side effect.






                                                                  share|improve this answer












                                                                  The changing/failing gravity is a floating point error in the universe simulation.



                                                                  Many existing simulations of varying complexity use floating point numbers for their calculations. For example, the Unity game engine uses a floating point number for gravity strength. In this case, the floating point number would be the lifetime or duration of the gravity effect. It could even be tracking the lifetime of the universe, and the gravity is just a side effect.







                                                                  share|improve this answer












                                                                  share|improve this answer



                                                                  share|improve this answer










                                                                  answered Aug 10 at 16:51









                                                                  Jarod S

                                                                  1




                                                                  1











                                                                  • Welcome to Worldbuilding, Jarod S! If you have a moment, please take the tour and visit the help center to learn more about the site. You may also find Worldbuilding Meta and The Sandbox (both of which require 5 rep to post on) useful. Here is a meta post on the culture and style of Worldbuilding.SE, just to help you understand our scope and methods, and how we do things here. Have fun!
                                                                    – Gryphon
                                                                    Aug 10 at 17:13
















                                                                  • Welcome to Worldbuilding, Jarod S! If you have a moment, please take the tour and visit the help center to learn more about the site. You may also find Worldbuilding Meta and The Sandbox (both of which require 5 rep to post on) useful. Here is a meta post on the culture and style of Worldbuilding.SE, just to help you understand our scope and methods, and how we do things here. Have fun!
                                                                    – Gryphon
                                                                    Aug 10 at 17:13















                                                                  Welcome to Worldbuilding, Jarod S! If you have a moment, please take the tour and visit the help center to learn more about the site. You may also find Worldbuilding Meta and The Sandbox (both of which require 5 rep to post on) useful. Here is a meta post on the culture and style of Worldbuilding.SE, just to help you understand our scope and methods, and how we do things here. Have fun!
                                                                  – Gryphon
                                                                  Aug 10 at 17:13




                                                                  Welcome to Worldbuilding, Jarod S! If you have a moment, please take the tour and visit the help center to learn more about the site. You may also find Worldbuilding Meta and The Sandbox (both of which require 5 rep to post on) useful. Here is a meta post on the culture and style of Worldbuilding.SE, just to help you understand our scope and methods, and how we do things here. Have fun!
                                                                  – Gryphon
                                                                  Aug 10 at 17:13










                                                                  up vote
                                                                  0
                                                                  down vote













                                                                  Earth isn't real. I'm borrowing an idea from fiction:



                                                                  Specifically, Mutineer's Moon, the first book of David Weber's Fifth Imperium series. They needed to hide a very large starship in our solar system--they took the surface off the Moon and put it on the starship, tossed the core into the sun and took the moon's place.



                                                                  Lets take this further--at some time in the past aliens replaced Earth with a hidden starship. The fakery is good enough that even sounding of the innards doesen't reveal that it's starship and not planet. However, the starship is nowhere near as massive as the planet, it had to use a gravity generator to make things look real.



                                                                  The maintenance system on the starship is failing. There are actually many gravity generators in the starship so a failure doesn't leave the planet coming apart, but now they are wearing out. As generators fail the others take up the load--but not quite evenly and the additional load on them speeds their own failure and once enough have failed they can't maintain 1g anymore.



                                                                  When all the generators are dead there will still be some gravity due to the mass of the ship and the mass of the crust that was laid over the ship, but it won't be anything like 1g and won't keep an atmosphere for the long term.






                                                                  share|improve this answer
























                                                                    up vote
                                                                    0
                                                                    down vote













                                                                    Earth isn't real. I'm borrowing an idea from fiction:



                                                                    Specifically, Mutineer's Moon, the first book of David Weber's Fifth Imperium series. They needed to hide a very large starship in our solar system--they took the surface off the Moon and put it on the starship, tossed the core into the sun and took the moon's place.



                                                                    Lets take this further--at some time in the past aliens replaced Earth with a hidden starship. The fakery is good enough that even sounding of the innards doesen't reveal that it's starship and not planet. However, the starship is nowhere near as massive as the planet, it had to use a gravity generator to make things look real.



                                                                    The maintenance system on the starship is failing. There are actually many gravity generators in the starship so a failure doesn't leave the planet coming apart, but now they are wearing out. As generators fail the others take up the load--but not quite evenly and the additional load on them speeds their own failure and once enough have failed they can't maintain 1g anymore.



                                                                    When all the generators are dead there will still be some gravity due to the mass of the ship and the mass of the crust that was laid over the ship, but it won't be anything like 1g and won't keep an atmosphere for the long term.






                                                                    share|improve this answer






















                                                                      up vote
                                                                      0
                                                                      down vote










                                                                      up vote
                                                                      0
                                                                      down vote









                                                                      Earth isn't real. I'm borrowing an idea from fiction:



                                                                      Specifically, Mutineer's Moon, the first book of David Weber's Fifth Imperium series. They needed to hide a very large starship in our solar system--they took the surface off the Moon and put it on the starship, tossed the core into the sun and took the moon's place.



                                                                      Lets take this further--at some time in the past aliens replaced Earth with a hidden starship. The fakery is good enough that even sounding of the innards doesen't reveal that it's starship and not planet. However, the starship is nowhere near as massive as the planet, it had to use a gravity generator to make things look real.



                                                                      The maintenance system on the starship is failing. There are actually many gravity generators in the starship so a failure doesn't leave the planet coming apart, but now they are wearing out. As generators fail the others take up the load--but not quite evenly and the additional load on them speeds their own failure and once enough have failed they can't maintain 1g anymore.



                                                                      When all the generators are dead there will still be some gravity due to the mass of the ship and the mass of the crust that was laid over the ship, but it won't be anything like 1g and won't keep an atmosphere for the long term.






                                                                      share|improve this answer












                                                                      Earth isn't real. I'm borrowing an idea from fiction:



                                                                      Specifically, Mutineer's Moon, the first book of David Weber's Fifth Imperium series. They needed to hide a very large starship in our solar system--they took the surface off the Moon and put it on the starship, tossed the core into the sun and took the moon's place.



                                                                      Lets take this further--at some time in the past aliens replaced Earth with a hidden starship. The fakery is good enough that even sounding of the innards doesen't reveal that it's starship and not planet. However, the starship is nowhere near as massive as the planet, it had to use a gravity generator to make things look real.



                                                                      The maintenance system on the starship is failing. There are actually many gravity generators in the starship so a failure doesn't leave the planet coming apart, but now they are wearing out. As generators fail the others take up the load--but not quite evenly and the additional load on them speeds their own failure and once enough have failed they can't maintain 1g anymore.



                                                                      When all the generators are dead there will still be some gravity due to the mass of the ship and the mass of the crust that was laid over the ship, but it won't be anything like 1g and won't keep an atmosphere for the long term.







                                                                      share|improve this answer












                                                                      share|improve this answer



                                                                      share|improve this answer










                                                                      answered Aug 11 at 3:17









                                                                      Loren Pechtel

                                                                      17.5k2154




                                                                      17.5k2154




















                                                                          up vote
                                                                          0
                                                                          down vote













                                                                          A great many people have independently considered the idea that gravity doesn't attract, it repels. The attraction observed between neighboring bodies is do to partial shielding of the bodies by one another. The shielding is small and proportional to mass. This doesn't actually work if you dig into the details, but you have to dig pretty deeply. The advantage now is that the source of gravitational pressure can be entirely divorced from mass and located a very long way away. Enabling this view of gravity would open up all kinds of options. It would seem that you are on the cusp of some event horizon that is about to isolate our corner of the universe from it's source of gravitination.






                                                                          share|improve this answer




















                                                                          • Do you have anything to back up your statement that gravity is repulsive? Because gravitational attraction between bodies has been measured in a lab
                                                                            – L.Dutch♦
                                                                            Aug 11 at 19:49










                                                                          • @L.Dutch I said it doesn't work. I know it doesn't work. But it is far from obvious that it doesn't work, and it opens up a lot of options. Hence "enabling this view ..."
                                                                            – Phil Sweet
                                                                            Aug 12 at 16:05














                                                                          up vote
                                                                          0
                                                                          down vote













                                                                          A great many people have independently considered the idea that gravity doesn't attract, it repels. The attraction observed between neighboring bodies is do to partial shielding of the bodies by one another. The shielding is small and proportional to mass. This doesn't actually work if you dig into the details, but you have to dig pretty deeply. The advantage now is that the source of gravitational pressure can be entirely divorced from mass and located a very long way away. Enabling this view of gravity would open up all kinds of options. It would seem that you are on the cusp of some event horizon that is about to isolate our corner of the universe from it's source of gravitination.






                                                                          share|improve this answer




















                                                                          • Do you have anything to back up your statement that gravity is repulsive? Because gravitational attraction between bodies has been measured in a lab
                                                                            – L.Dutch♦
                                                                            Aug 11 at 19:49










                                                                          • @L.Dutch I said it doesn't work. I know it doesn't work. But it is far from obvious that it doesn't work, and it opens up a lot of options. Hence "enabling this view ..."
                                                                            – Phil Sweet
                                                                            Aug 12 at 16:05












                                                                          up vote
                                                                          0
                                                                          down vote










                                                                          up vote
                                                                          0
                                                                          down vote









                                                                          A great many people have independently considered the idea that gravity doesn't attract, it repels. The attraction observed between neighboring bodies is do to partial shielding of the bodies by one another. The shielding is small and proportional to mass. This doesn't actually work if you dig into the details, but you have to dig pretty deeply. The advantage now is that the source of gravitational pressure can be entirely divorced from mass and located a very long way away. Enabling this view of gravity would open up all kinds of options. It would seem that you are on the cusp of some event horizon that is about to isolate our corner of the universe from it's source of gravitination.






                                                                          share|improve this answer












                                                                          A great many people have independently considered the idea that gravity doesn't attract, it repels. The attraction observed between neighboring bodies is do to partial shielding of the bodies by one another. The shielding is small and proportional to mass. This doesn't actually work if you dig into the details, but you have to dig pretty deeply. The advantage now is that the source of gravitational pressure can be entirely divorced from mass and located a very long way away. Enabling this view of gravity would open up all kinds of options. It would seem that you are on the cusp of some event horizon that is about to isolate our corner of the universe from it's source of gravitination.







                                                                          share|improve this answer












                                                                          share|improve this answer



                                                                          share|improve this answer










                                                                          answered Aug 11 at 18:09









                                                                          Phil

                                                                          1




                                                                          1











                                                                          • Do you have anything to back up your statement that gravity is repulsive? Because gravitational attraction between bodies has been measured in a lab
                                                                            – L.Dutch♦
                                                                            Aug 11 at 19:49










                                                                          • @L.Dutch I said it doesn't work. I know it doesn't work. But it is far from obvious that it doesn't work, and it opens up a lot of options. Hence "enabling this view ..."
                                                                            – Phil Sweet
                                                                            Aug 12 at 16:05
















                                                                          • Do you have anything to back up your statement that gravity is repulsive? Because gravitational attraction between bodies has been measured in a lab
                                                                            – L.Dutch♦
                                                                            Aug 11 at 19:49










                                                                          • @L.Dutch I said it doesn't work. I know it doesn't work. But it is far from obvious that it doesn't work, and it opens up a lot of options. Hence "enabling this view ..."
                                                                            – Phil Sweet
                                                                            Aug 12 at 16:05















                                                                          Do you have anything to back up your statement that gravity is repulsive? Because gravitational attraction between bodies has been measured in a lab
                                                                          – L.Dutch♦
                                                                          Aug 11 at 19:49




                                                                          Do you have anything to back up your statement that gravity is repulsive? Because gravitational attraction between bodies has been measured in a lab
                                                                          – L.Dutch♦
                                                                          Aug 11 at 19:49












                                                                          @L.Dutch I said it doesn't work. I know it doesn't work. But it is far from obvious that it doesn't work, and it opens up a lot of options. Hence "enabling this view ..."
                                                                          – Phil Sweet
                                                                          Aug 12 at 16:05




                                                                          @L.Dutch I said it doesn't work. I know it doesn't work. But it is far from obvious that it doesn't work, and it opens up a lot of options. Hence "enabling this view ..."
                                                                          – Phil Sweet
                                                                          Aug 12 at 16:05










                                                                          up vote
                                                                          0
                                                                          down vote













                                                                          Alien Transportation System



                                                                          OK, this one sounds a bit "space opera"-ish, but you can probably dress it up and make it pretty.



                                                                          Aliens from an alternate dimension/universe/what-have-you use "our" dimension/universe/etc as a convenient way to travel great distances in "their" dimension, because the mapping between the dimensions is such that traveling (a short distance) in "our" dimension is equivalent to traveling (a very very long distance) in theirs. Jumping from a "small" dimension (theirs) to a "big" dimension (ours) requires very little energy - but traveling from a "big" dimension to a "small" dimension requires A Lot of energy - and the energy (and thus the mass) is removed from the local area near the jump site and transferred to the "small" dimension. And, again unfortunately, the Sol system just happens to be conveniently located for their use - so they're sucking the energy/mass out of our area. Oh, BTW, that means the sun is going to go nova. Sorry about your luck, humans - BUT as a consolation prize the aliens have offered each and every human a set of genuine Alien Ginsu Steak Knives! They slice! They dice! They make your star blow up! Wooooo!






                                                                          share|improve this answer
























                                                                            up vote
                                                                            0
                                                                            down vote













                                                                            Alien Transportation System



                                                                            OK, this one sounds a bit "space opera"-ish, but you can probably dress it up and make it pretty.



                                                                            Aliens from an alternate dimension/universe/what-have-you use "our" dimension/universe/etc as a convenient way to travel great distances in "their" dimension, because the mapping between the dimensions is such that traveling (a short distance) in "our" dimension is equivalent to traveling (a very very long distance) in theirs. Jumping from a "small" dimension (theirs) to a "big" dimension (ours) requires very little energy - but traveling from a "big" dimension to a "small" dimension requires A Lot of energy - and the energy (and thus the mass) is removed from the local area near the jump site and transferred to the "small" dimension. And, again unfortunately, the Sol system just happens to be conveniently located for their use - so they're sucking the energy/mass out of our area. Oh, BTW, that means the sun is going to go nova. Sorry about your luck, humans - BUT as a consolation prize the aliens have offered each and every human a set of genuine Alien Ginsu Steak Knives! They slice! They dice! They make your star blow up! Wooooo!






                                                                            share|improve this answer






















                                                                              up vote
                                                                              0
                                                                              down vote










                                                                              up vote
                                                                              0
                                                                              down vote









                                                                              Alien Transportation System



                                                                              OK, this one sounds a bit "space opera"-ish, but you can probably dress it up and make it pretty.



                                                                              Aliens from an alternate dimension/universe/what-have-you use "our" dimension/universe/etc as a convenient way to travel great distances in "their" dimension, because the mapping between the dimensions is such that traveling (a short distance) in "our" dimension is equivalent to traveling (a very very long distance) in theirs. Jumping from a "small" dimension (theirs) to a "big" dimension (ours) requires very little energy - but traveling from a "big" dimension to a "small" dimension requires A Lot of energy - and the energy (and thus the mass) is removed from the local area near the jump site and transferred to the "small" dimension. And, again unfortunately, the Sol system just happens to be conveniently located for their use - so they're sucking the energy/mass out of our area. Oh, BTW, that means the sun is going to go nova. Sorry about your luck, humans - BUT as a consolation prize the aliens have offered each and every human a set of genuine Alien Ginsu Steak Knives! They slice! They dice! They make your star blow up! Wooooo!






                                                                              share|improve this answer












                                                                              Alien Transportation System



                                                                              OK, this one sounds a bit "space opera"-ish, but you can probably dress it up and make it pretty.



                                                                              Aliens from an alternate dimension/universe/what-have-you use "our" dimension/universe/etc as a convenient way to travel great distances in "their" dimension, because the mapping between the dimensions is such that traveling (a short distance) in "our" dimension is equivalent to traveling (a very very long distance) in theirs. Jumping from a "small" dimension (theirs) to a "big" dimension (ours) requires very little energy - but traveling from a "big" dimension to a "small" dimension requires A Lot of energy - and the energy (and thus the mass) is removed from the local area near the jump site and transferred to the "small" dimension. And, again unfortunately, the Sol system just happens to be conveniently located for their use - so they're sucking the energy/mass out of our area. Oh, BTW, that means the sun is going to go nova. Sorry about your luck, humans - BUT as a consolation prize the aliens have offered each and every human a set of genuine Alien Ginsu Steak Knives! They slice! They dice! They make your star blow up! Wooooo!







                                                                              share|improve this answer












                                                                              share|improve this answer



                                                                              share|improve this answer










                                                                              answered Aug 12 at 16:18









                                                                              Bob Jarvis

                                                                              25137




                                                                              25137




















                                                                                  up vote
                                                                                  -1
                                                                                  down vote













                                                                                  No



                                                                                  Gravity doesn't "fail", any more than the other fundamental forces fail. If they could, we would live in an entirely different universe. If any of the forces "started to fail", so many catastrophic things would happen all at once that there would be no grace period.






                                                                                  share|improve this answer
















                                                                                  • 3




                                                                                    A brief look at the history of science yields that long-held theories about the way the universe works have a broad tendency toward failure. We're constantly revising our understanding of the universe because we're frequently wrong. Gravity is just a theory which we hold to be true because we've never observed it failing, but without a solid understanding of how much probabilistic weight a single instance of nonfailure yields, our lives are too short to develop the kind of certainty this answer reflects; at least not without some higher source of absolute knowledge.
                                                                                    – boxcartenant
                                                                                    Aug 9 at 19:31







                                                                                  • 1




                                                                                    I'm with @boxcartenant. I was regularly frustrated by a young woman taking nursing classes because she would spout today's medical understanding and complete her statement with the phrase, "and that's a medical fact!" She's 20 something. I'm 50 something. I've lived long enough to know some of the so-called medical facts she's spouting didn't even exist at one point or changed to accomodate recent research. Our understanding of just about everything is hardly complete. To paraphrase the movie Volcano, "we're scientists, 'certainty' is a big word."
                                                                                    – JBH
                                                                                    Aug 9 at 19:51






                                                                                  • 1




                                                                                    @boxcartenant: Long-held theories are being revised, not invalidated. Newton's Law of Gravity still works for most purposes. There have been some big changes, but they tend to explain new things. Photons explained the previously unexplained cases of the photoelectric effect and black-body radiation, but optics wasn't really affected. We know that physical laws are pretty much the same all over the observable Universe.
                                                                                    – David Thornley
                                                                                    Aug 9 at 20:18






                                                                                  • 1




                                                                                    @DavidThornley Good feedback. I think, though, a revision is essentially a partial invalidation. We'll never completely invalidate gravity, because we can plainly see that things are falling, but we might revise our understanding of it so significantly that it would be unrecognizable against its current form; so while the laws might be the same everywhere, their application may be entirely different from what we currently suppose.
                                                                                    – boxcartenant
                                                                                    Aug 9 at 20:38






                                                                                  • 1




                                                                                    This is not a reality check.
                                                                                    – Theraot
                                                                                    Aug 10 at 1:23














                                                                                  up vote
                                                                                  -1
                                                                                  down vote













                                                                                  No



                                                                                  Gravity doesn't "fail", any more than the other fundamental forces fail. If they could, we would live in an entirely different universe. If any of the forces "started to fail", so many catastrophic things would happen all at once that there would be no grace period.






                                                                                  share|improve this answer
















                                                                                  • 3




                                                                                    A brief look at the history of science yields that long-held theories about the way the universe works have a broad tendency toward failure. We're constantly revising our understanding of the universe because we're frequently wrong. Gravity is just a theory which we hold to be true because we've never observed it failing, but without a solid understanding of how much probabilistic weight a single instance of nonfailure yields, our lives are too short to develop the kind of certainty this answer reflects; at least not without some higher source of absolute knowledge.
                                                                                    – boxcartenant
                                                                                    Aug 9 at 19:31







                                                                                  • 1




                                                                                    I'm with @boxcartenant. I was regularly frustrated by a young woman taking nursing classes because she would spout today's medical understanding and complete her statement with the phrase, "and that's a medical fact!" She's 20 something. I'm 50 something. I've lived long enough to know some of the so-called medical facts she's spouting didn't even exist at one point or changed to accomodate recent research. Our understanding of just about everything is hardly complete. To paraphrase the movie Volcano, "we're scientists, 'certainty' is a big word."
                                                                                    – JBH
                                                                                    Aug 9 at 19:51






                                                                                  • 1




                                                                                    @boxcartenant: Long-held theories are being revised, not invalidated. Newton's Law of Gravity still works for most purposes. There have been some big changes, but they tend to explain new things. Photons explained the previously unexplained cases of the photoelectric effect and black-body radiation, but optics wasn't really affected. We know that physical laws are pretty much the same all over the observable Universe.
                                                                                    – David Thornley
                                                                                    Aug 9 at 20:18






                                                                                  • 1




                                                                                    @DavidThornley Good feedback. I think, though, a revision is essentially a partial invalidation. We'll never completely invalidate gravity, because we can plainly see that things are falling, but we might revise our understanding of it so significantly that it would be unrecognizable against its current form; so while the laws might be the same everywhere, their application may be entirely different from what we currently suppose.
                                                                                    – boxcartenant
                                                                                    Aug 9 at 20:38






                                                                                  • 1




                                                                                    This is not a reality check.
                                                                                    – Theraot
                                                                                    Aug 10 at 1:23












                                                                                  up vote
                                                                                  -1
                                                                                  down vote










                                                                                  up vote
                                                                                  -1
                                                                                  down vote









                                                                                  No



                                                                                  Gravity doesn't "fail", any more than the other fundamental forces fail. If they could, we would live in an entirely different universe. If any of the forces "started to fail", so many catastrophic things would happen all at once that there would be no grace period.






                                                                                  share|improve this answer












                                                                                  No



                                                                                  Gravity doesn't "fail", any more than the other fundamental forces fail. If they could, we would live in an entirely different universe. If any of the forces "started to fail", so many catastrophic things would happen all at once that there would be no grace period.







                                                                                  share|improve this answer












                                                                                  share|improve this answer



                                                                                  share|improve this answer










                                                                                  answered Aug 9 at 19:11









                                                                                  jdunlop

                                                                                  5,2521934




                                                                                  5,2521934







                                                                                  • 3




                                                                                    A brief look at the history of science yields that long-held theories about the way the universe works have a broad tendency toward failure. We're constantly revising our understanding of the universe because we're frequently wrong. Gravity is just a theory which we hold to be true because we've never observed it failing, but without a solid understanding of how much probabilistic weight a single instance of nonfailure yields, our lives are too short to develop the kind of certainty this answer reflects; at least not without some higher source of absolute knowledge.
                                                                                    – boxcartenant
                                                                                    Aug 9 at 19:31







                                                                                  • 1




                                                                                    I'm with @boxcartenant. I was regularly frustrated by a young woman taking nursing classes because she would spout today's medical understanding and complete her statement with the phrase, "and that's a medical fact!" She's 20 something. I'm 50 something. I've lived long enough to know some of the so-called medical facts she's spouting didn't even exist at one point or changed to accomodate recent research. Our understanding of just about everything is hardly complete. To paraphrase the movie Volcano, "we're scientists, 'certainty' is a big word."
                                                                                    – JBH
                                                                                    Aug 9 at 19:51






                                                                                  • 1




                                                                                    @boxcartenant: Long-held theories are being revised, not invalidated. Newton's Law of Gravity still works for most purposes. There have been some big changes, but they tend to explain new things. Photons explained the previously unexplained cases of the photoelectric effect and black-body radiation, but optics wasn't really affected. We know that physical laws are pretty much the same all over the observable Universe.
                                                                                    – David Thornley
                                                                                    Aug 9 at 20:18






                                                                                  • 1




                                                                                    @DavidThornley Good feedback. I think, though, a revision is essentially a partial invalidation. We'll never completely invalidate gravity, because we can plainly see that things are falling, but we might revise our understanding of it so significantly that it would be unrecognizable against its current form; so while the laws might be the same everywhere, their application may be entirely different from what we currently suppose.
                                                                                    – boxcartenant
                                                                                    Aug 9 at 20:38






                                                                                  • 1




                                                                                    This is not a reality check.
                                                                                    – Theraot
                                                                                    Aug 10 at 1:23












                                                                                  • 3




                                                                                    A brief look at the history of science yields that long-held theories about the way the universe works have a broad tendency toward failure. We're constantly revising our understanding of the universe because we're frequently wrong. Gravity is just a theory which we hold to be true because we've never observed it failing, but without a solid understanding of how much probabilistic weight a single instance of nonfailure yields, our lives are too short to develop the kind of certainty this answer reflects; at least not without some higher source of absolute knowledge.
                                                                                    – boxcartenant
                                                                                    Aug 9 at 19:31







                                                                                  • 1




                                                                                    I'm with @boxcartenant. I was regularly frustrated by a young woman taking nursing classes because she would spout today's medical understanding and complete her statement with the phrase, "and that's a medical fact!" She's 20 something. I'm 50 something. I've lived long enough to know some of the so-called medical facts she's spouting didn't even exist at one point or changed to accomodate recent research. Our understanding of just about everything is hardly complete. To paraphrase the movie Volcano, "we're scientists, 'certainty' is a big word."
                                                                                    – JBH
                                                                                    Aug 9 at 19:51






                                                                                  • 1




                                                                                    @boxcartenant: Long-held theories are being revised, not invalidated. Newton's Law of Gravity still works for most purposes. There have been some big changes, but they tend to explain new things. Photons explained the previously unexplained cases of the photoelectric effect and black-body radiation, but optics wasn't really affected. We know that physical laws are pretty much the same all over the observable Universe.
                                                                                    – David Thornley
                                                                                    Aug 9 at 20:18






                                                                                  • 1




                                                                                    @DavidThornley Good feedback. I think, though, a revision is essentially a partial invalidation. We'll never completely invalidate gravity, because we can plainly see that things are falling, but we might revise our understanding of it so significantly that it would be unrecognizable against its current form; so while the laws might be the same everywhere, their application may be entirely different from what we currently suppose.
                                                                                    – boxcartenant
                                                                                    Aug 9 at 20:38






                                                                                  • 1




                                                                                    This is not a reality check.
                                                                                    – Theraot
                                                                                    Aug 10 at 1:23







                                                                                  3




                                                                                  3




                                                                                  A brief look at the history of science yields that long-held theories about the way the universe works have a broad tendency toward failure. We're constantly revising our understanding of the universe because we're frequently wrong. Gravity is just a theory which we hold to be true because we've never observed it failing, but without a solid understanding of how much probabilistic weight a single instance of nonfailure yields, our lives are too short to develop the kind of certainty this answer reflects; at least not without some higher source of absolute knowledge.
                                                                                  – boxcartenant
                                                                                  Aug 9 at 19:31





                                                                                  A brief look at the history of science yields that long-held theories about the way the universe works have a broad tendency toward failure. We're constantly revising our understanding of the universe because we're frequently wrong. Gravity is just a theory which we hold to be true because we've never observed it failing, but without a solid understanding of how much probabilistic weight a single instance of nonfailure yields, our lives are too short to develop the kind of certainty this answer reflects; at least not without some higher source of absolute knowledge.
                                                                                  – boxcartenant
                                                                                  Aug 9 at 19:31





                                                                                  1




                                                                                  1




                                                                                  I'm with @boxcartenant. I was regularly frustrated by a young woman taking nursing classes because she would spout today's medical understanding and complete her statement with the phrase, "and that's a medical fact!" She's 20 something. I'm 50 something. I've lived long enough to know some of the so-called medical facts she's spouting didn't even exist at one point or changed to accomodate recent research. Our understanding of just about everything is hardly complete. To paraphrase the movie Volcano, "we're scientists, 'certainty' is a big word."
                                                                                  – JBH
                                                                                  Aug 9 at 19:51




                                                                                  I'm with @boxcartenant. I was regularly frustrated by a young woman taking nursing classes because she would spout today's medical understanding and complete her statement with the phrase, "and that's a medical fact!" She's 20 something. I'm 50 something. I've lived long enough to know some of the so-called medical facts she's spouting didn't even exist at one point or changed to accomodate recent research. Our understanding of just about everything is hardly complete. To paraphrase the movie Volcano, "we're scientists, 'certainty' is a big word."
                                                                                  – JBH
                                                                                  Aug 9 at 19:51




                                                                                  1




                                                                                  1




                                                                                  @boxcartenant: Long-held theories are being revised, not invalidated. Newton's Law of Gravity still works for most purposes. There have been some big changes, but they tend to explain new things. Photons explained the previously unexplained cases of the photoelectric effect and black-body radiation, but optics wasn't really affected. We know that physical laws are pretty much the same all over the observable Universe.
                                                                                  – David Thornley
                                                                                  Aug 9 at 20:18




                                                                                  @boxcartenant: Long-held theories are being revised, not invalidated. Newton's Law of Gravity still works for most purposes. There have been some big changes, but they tend to explain new things. Photons explained the previously unexplained cases of the photoelectric effect and black-body radiation, but optics wasn't really affected. We know that physical laws are pretty much the same all over the observable Universe.
                                                                                  – David Thornley
                                                                                  Aug 9 at 20:18




                                                                                  1




                                                                                  1




                                                                                  @DavidThornley Good feedback. I think, though, a revision is essentially a partial invalidation. We'll never completely invalidate gravity, because we can plainly see that things are falling, but we might revise our understanding of it so significantly that it would be unrecognizable against its current form; so while the laws might be the same everywhere, their application may be entirely different from what we currently suppose.
                                                                                  – boxcartenant
                                                                                  Aug 9 at 20:38




                                                                                  @DavidThornley Good feedback. I think, though, a revision is essentially a partial invalidation. We'll never completely invalidate gravity, because we can plainly see that things are falling, but we might revise our understanding of it so significantly that it would be unrecognizable against its current form; so while the laws might be the same everywhere, their application may be entirely different from what we currently suppose.
                                                                                  – boxcartenant
                                                                                  Aug 9 at 20:38




                                                                                  1




                                                                                  1




                                                                                  This is not a reality check.
                                                                                  – Theraot
                                                                                  Aug 10 at 1:23




                                                                                  This is not a reality check.
                                                                                  – Theraot
                                                                                  Aug 10 at 1:23

















                                                                                   

                                                                                  draft saved


                                                                                  draft discarded















































                                                                                   


                                                                                  draft saved


                                                                                  draft discarded














                                                                                  StackExchange.ready(
                                                                                  function ()
                                                                                  StackExchange.openid.initPostLogin('.new-post-login', 'https%3a%2f%2fworldbuilding.stackexchange.com%2fquestions%2f120899%2fplausible-hypothesis-on-why-gravity-might-fail-on-earth%23new-answer', 'question_page');

                                                                                  );

                                                                                  Post as a guest













































































                                                                                  Comments

                                                                                  Popular posts from this blog

                                                                                  Long meetings (6-7 hours a day): Being “babysat” by supervisor

                                                                                  Is the Concept of Multiple Fantasy Races Scientifically Flawed? [closed]

                                                                                  Confectionery